You are on page 1of 74

MATERI PENGANTAR

OLIMPIADE SAINS NASIONAL

BIDANG MATEMATIKA SMA

DISUSUN OLEH:

TIM PEMBINA OLIMPIADE MATEMATIKA

TIM OLIMPIADE MATEMATIKA INDONESIA


Juli 2009
KATA PENGANTAR

Olimpiade Sains Nasional (OSN) merupakan kegiatan yang dilaksanakan


secara rutin setiap tahun. Selain sebagai ajang kejuaraan nasional, OSN juga
bertujuan untuk mencari calon peserta yang akan dibina dan berlaga di olimpiade
sains tingkat internasional. Untuk memberikan arah pembinaan bagi peserta OSN,
terutama peserta bidang matematika SMA, maka disusunlah materi ini.

Materi yang kami susun ini merupakan draf awal yang kami kembangkan
dari silabus OSN Bidang Matematika SMA yang sudah beredar. Draf ini jauh dari
lengkap seperti yang tercantum dalam silabus. Oleh karena itu para peserta OSN
dan para pembina masih perlu mencari pelengkapnya dari berbagai sumber.

Terakhir, kami berharap bahwa dengan adanya draf ini persiapan ke OSN
tahun ini bisa lebih baik dibanding tahun-tahun sebelumnya. Pada akhirnya akan
berdampak kepada kualitas calon peserta IMO tahun depan.

Jakarta, 1 Juli 2009

Tim Pembina Olimpiade Matematika


DAFTAR ISI

Aljabar 1

Teori Bilangan 14

Geometri 36

Kombinatorika 60
ALJABAR
Hery Susanto
Jurusan Matematika FMIPA UM
Jl. Surabaya 6 Malang 65145
Email: herysusanto@telkom.net

Aljabar merupakan salah satu materi pokok dalam Olimpiade Matematika Internasional (IMO),
disamping geometri, ilmu bilangan, dan kombinatorik. Oleh karena itu, aljabar menjadi salah satu
materi wajib di Olimpiade Sains Nasional (OSN) Bidang Matematika SMA. Para peserta OSN
Bidang Matematika SMA sudah mendapatkan materi aljabar sejak masih duduk di Sekolah Dasar.
Menurut pengamatan penulis, sampai dengan di tingkat SMA para peserta sudah cukup banyak
mendapat materi yang berkaitan dengan aljabar. Yang kurang adalah berlatih menerapkan teori
yang telah didapat itu dalam soal-soal yang berbentuk pemecahan masalah. Di sini penulis men-
coba memilihkan materi-materi aljabar yang kira-kira mengarah ke materi olimpiade matematika.
Penulis tidak banyak menyajikan materi yang berkaitan dengan teori tetapi lebih banyak dengan
pendekatan strategi pemecahan masalah.
Materi yang dibahas di sini hanya sebagian dari materi yang ada pada silabus OSN Bidang
Matematika SMA. Kami berharap peserta merujuk kepada silabus OSN yang sudah beredar. Di
bagian akhir diberikan beberapa soal latihan dan diharapkan dapat dipakai untuk memacu dalam
mengembangkan kemampuan pemecahan masalah.
Pada akhirnya penulis berharap agar para peserta OSN Bidang Matematika SMA dapat meru-
juk materi yang tercantum dalam silabus OSN dan membaca buku-buku tentang pemecahan
masalah untuk mengembangkan kemampuan pemecahan masalah ke tingkat yang lebih tinggi.

1 Sistem Bilangan Real


Misalkan N, Z, Q, dan R berturut-turut menyatakan himpunan bilangan asli, himpunan bilangan
bulat, himpunan bilangan rasional, dan himpunan bilangan real. Masing-masing himpunan ini
dilengkapi dengan operasi tambah dan operasi kali disebut sistem bilangan. Sistem ini biasa di-
tulis notasi himpunan beserta operasinya. Sebagai contoh sistem bilangan real ditulis (R, +, ×).
Selanjutnya cukup ditulis notasi himpunannya saja, yaitu R.
Berikut akan dibahas dua aksioma yang berkaitan dengan sistem bilangan real, yaitu aksioma
lapangan dan aksioma urutan.

1.1 Aksioma Lapangan


1. Sifat asosiatif, yaitu untuk setiap a, b, c di R berlaku

(a) (a + b) + c = a + (b + c),
(b) (ab)c = a(bc).

2. Sifat komutatif, yaitu untuk setiap a, b, c di R berlaku

1
(a) a + b = b + a,
(b) ab = ba.

3. Eksistensi unsur identitas, yaitu

(a) Terdapat 0 di R yang memenuhi


a+0=a
untuk semua a di R.
(b) Terdapat 1 di R dan 1 6= 0 yang memenuhi

a1 = a

untuk semua a di R.

4. Eksistensi unsur invers, yaitu

(a) Untuk masing-masing a di R terdapat −a di R yang memenuhi

a + (−a) = 0.

(b) Untuk masing-masing a di R yang tidak nol terdapat a−1 di R yang memenuhi

aa−1 = 1.

5. Sifat distributif, yaitu untuk setiap a, b, c di R berlaku

a(b + c) = ab + ac.

Sebagai kosekuensi dari sifat-sifat pada aksioma lapangan di atas, diperoleh sifat-sifat berikut
(yang dapat dianggap sebagai teorema).

1. −a dan a−1 yang memenuhi sifat 4 di atas adalah tunggal.

2. 0a = 0, untuk setiap a di R.

3. (−1)a = −a, untuk setiap a di R.

4. −(−a) = a, untuk setiap a di R.

5. (−a)(−b) = ab, untuk setiap a di R.


−1
6. a−1 = a, untuk setiap a di R yang tidak nol.

Sebagai suatu teorema, sifat-sifat di atas harus dibuktikan. Berikut diberikan beberapa contoh
pembuktian.

2
Bukti untuk 2. Ambil a ∈ R sebarang. Dengan menggunakan sifat identitas, invers, asosiatif,
dan distributif diperoleh

0a = 0 + 0a
= (−0a + 0a) + 0a
= −0a + (0a + 0a)
= −0a + (0 + 0)a
= −0a + 0a
= 0.

Karena a ∈ R sebarang, maka 0a = 0, untuk setiap a di R.

Bukti untuk 3. Ambil a ∈ R sebarang. Dengan menggunakan sifat identitas, invers, asosiatif,
distributif, dan sifat 2 di atas diperoleh

(−1)a = 0 + (−1)a
= (−a + a) + (−1)a
= −a + (a + (−1)a)
= −a + (1a + (−1)a)
= −a + (1 + (−1))a
= −a + 0a
= −a + 0
= −a.

Karena a ∈ R sebarang, maka (−1)a = −a, untuk setiap a di R.

Bukti keempat sifat yang lain digunakan sebagai latihan.

1.2 Aksioma Urutan


Terdapat himpunan P yang merupakan himpunan bagian dari himpunan bilangan real yang memenuhi
tiga sifat berikut.

(1) Untuk bilangan real x sebarang, berlaku salah satu dari: (i) x = 0, atau (ii) x ∈ P , atau (iii)
−x ∈ P .

(2) Jika x, y ∈ P , maka x + y ∈ P .

(3) Jika x, y ∈ P , maka xy ∈ P .

Sifat (1), (2), dan (3) di atas berturut-turut disebut sifat trikotomi, sifat ketertutupan operasi
tambah, dan sifat ketertutupan operasi kali. Himpunan P di atas disebut himpunan bilangan real
positif. Berikutnya ada kesepakatan bahwa notasi x > 0 digunakan jika x ∈ P . Dengan demikian
ketiga sifat di atas dapat ditulis ulang sebagai berikut.

(1’) Untuk bilangan real x sebarang, berlaku salah satu dari: (i) x = 0, atau (ii) x > 0, atau (iii)
−x > 0.

3
(2’) Jika x > 0 dan y > 0, maka x + y > 0.

(3’) Jika x > 0 dan y > 0, maka xy > 0.

Berikut didefinisikan relasi ”lebih besar dari” dan relasi ”lebih kecil dari” untuk dua bilangan
real. x dikatakan lebih besar dari y, dinotasikan x > y, jika x − y > 0. x dikatakan lebih kecil dari
y, dinotasikan x < y, jika y − x > 0. Dapat ditunjukkan bahwa x > y ekivalen dengan y < x.
Notasi x ≥ y, dibaca x lebih besar dari atau sama dengan y, digunakan jika x > y atau x = y.

Konsekuensi dari sifat-sifat di atas, diperoleh sifat-sifat berikut (yang dapat dianggap sebagai
teorema).

1. Untuk setiap pasang bilangan real a dan b pasti berlaku salah satu dari a < b, atau a = b,
atau a > b.

2. Jika a < b dan b < c, maka a < c.

3. Jika a < b, maka a + c < b + c.

4. Jika a < b dan c > 0, maka ac < bc.

5. Jika a > 0 dan b > 0, maka ab > 0.

6. Jika a < b dan c < 0, maka ac > bc.

7. Untuk setiap a di R berlaku a2 ≥ 0. Selanjutnya, a2 = 0 jika dan hanya jika a = 0.


1
8. Jika a > b > 0, maka a < 1b .
a
9. Jika a > b > 0 dan c > d > 0, maka ac > bd dan d > cb .

10. Jika a > b, maka an > bn untuk n bilangan asli ganjil.

11. Jika a > b > 0, maka an > bn untuk n bilangan asli.

Bukti sifat-sifat di atas digunakan sebagai latihan.

2 Ketaksamaan
Sifat-sifat yang berkaitan dengan aksioma urutan banyak digunakan pada masalah-masalah yang
berkaitan dengan ketaksamaan. Pembahasan ketaksamaan berikut menggunakan pendekatan pe-
mecahan masalah dengan beberapa contoh.

Contoh 1 Tunjukkan bahwa a + a1 ≥ 2 untuk setiap bilangan real a > 0, dan akan merupakan
kesamaan jika dan hanya jika a = 1.

Penyelesaian: Untuk setiap bilangan real a berlaku

a2 − 2a + 1 = (a − 1)2 ≥ 0.

4
Sehingga a2 + 1 ≥ 2a. Karena a > 0 maka
1
a+ ≥ 2.
a
Selanjutnya
1
a+ = 2 ⇔ a2 + 1 = 2a ⇔ a2 − 2a + 1 = 0 ⇔ (a − 1)2 = 0 ⇔ a = 1.
a
Contoh 2 Misalkan a, b, dan c adalah bilangan real positif dan a + b + c = 1. Tunjukkan bahwa
1 1 1
+ + ≥ 9.
a b c
Penyelesaian: Dari a + b + c = 1 diperoleh
1 b c
= 1+ +
a a a
1 a c
= +1+
b b b
1 a b
= + +1
c c c
Sehingga
    
1 1 1 b c a c a b
+ + = 1+ + + +1+ + + +1
a b c a a b b c c
    
a b a c c b
= + + + + + +3
b a c a b c
a b a c c b
Menurut Contoh 1, b + a ≥ 2, c + a ≥ 2, b + c ≥ 2. Oleh karena itu

1 1 1
+ + ≥ 2 + 2 + 2 + 3 = 9.
a b c

Contoh 3 Buktikan bahwa jika a dan b bilangan real positif maka


r
a2 + b2 a+b √ 2
≥ ≥ ab ≥ 1 1 .
2 2 a + b

Selanjutnya, ketaksamaan ini akan berlaku sebagai kesamaan jika dan hanya jika a = b.

Bukti: Perhatikan bahwa


a2 − 2ab + b2 = (a − b)2 ≥ 0. (1)
Hal ini ekivalen dengan
a2 + b2 ≥ 2ab. (2)
Ditambah dengan a2 + b2 untuk kedua ruas, Ketaksamaan (2) ekivalen dengan

2 a2 + b2 ≥ a2 + 2ab + b2 = (a + b)2 ,


5
yang ekivalen dengan r
a2 + b2 a+b
≥ . (3)
2 2
Perhatikan bahwa Ketaksamaan (3) merupakan kesamaan jika dan hanya jika Ketaksamaan (1)
merupakan kesamaan. Hal ini terjadi jika dan hanya jika a = b. √

Dengan mengganti a dan b pada Ketaksamaan (2) berturut-turut dengan a dan b diperoleh

a + b ≥ 2 ab,

yang ekivalen dengan


a+b √
≥ ab. (4)
2
√ √
Dengan demikian, Ketaksamaan (4) merupakan kesamaan jika dan hanya jika a= b, atau a = b.
Ketaksamaan (4) ekivalen dengan
2 1
≤√ ,
a+b ab
yang ekivalen dengan
2 2 1 √
1 1 = ab ≤ √ ab = ab. (5)
a + b
a+b ab
Selanjutnya ketaksamaan (5) merupakan kesamaan jika dan hanya jika a = b.
Dari Ketaksamaan (3), (4), dan (5) diperoleh
r
a2 + b2 a+b √ 2
≥ ≥ ab ≥ 1 1 .
2 2 a + b

Selanjutnya, ketaksamaan di atas akan merupakan kesamaan jika dan hanya jika a = b. 
q
2 2 √
Untuk bilangan real positif a dan b, a +b a+b
2 , 2 ,
2
ab, dan 1 + 1 pada contoh di atas berturut-
a b
turut disebut rataan kuadrat (QM), rataan aritmatika (AM), rataan geometri (GM), dan rataan
harmonik (HM) dari a dan b. Dengan demikian, untuk bilangan real positif a dan b kita mempunyai
QM ≥ AM ≥ GM ≥ HM, selanjutnya QM = AM = GM = HM jika dan hanya jika a = b.
Rataan kuadrat, rataan aritmatika, rataan geometri, dan rataan harmonik dari n bilangan real
positif a1 , a2 , . . . , an berturut-turut adalah
r
a21 + a22 + . . . + a2n
QM =
n
a1 + a2 + . . . + an
AM = ,
√ n
GM = n a1 a2 . . . an , dan
n
HM = 1 1 1 .
a1 + a2 + . . . + an

Teorema berikut merupakan perumuman dari Contoh 3.

Teorema 1 Jika QM, AM, GM, dan HM berturut-turut menyatakan rataan kuarat, rataan arit-
matika, rataan geoetri, dan rataan harmonik dari bilangan real positif a1 , a2 , . . . , an , maka QM ≥
AM ≥ GM ≥ HM . Selanjutnya, ketaksamaan ini akan berlaku sebagai kesamaan jika dan hanya
jika a1 = a2 = . . . = an .

6
Contoh 4 Contoh 2 dikerjakan dengan menggunakan ketaksamaan AM-HM.

Menurut ketaksamaan AM-HM, kita mempunyai


3 a+b+c 1
1 1 1 ≤ = .
a + b + c
3 3
1 1 1 1 1 1
Karena a + b + c > 0 maka a + b + c ≥ 9.

Contoh 5 Misalkan a, b, dan c adalah bilangan real positif yang memenuhi (1+a)(1+b)(1+c) = 8.
Buktikan bahwa abc ≤ 1. Selanjutnya tentukan kapan kesamaan terjadi.

Penyelesaian: Dari yang diketahui diperoleh

8 = (1 + a)(1 + b)(1 + c) = 1 + (a + b + c) + (ab + bc + ca) + abc. (6)

Menurut ketaksamaan AM-GM,


1 2
a + b + c ≥ 3(abc) 3 dan ab + bc + ca ≥ 3(abc) 3 , (7)

masing-masing ketaksamaan di atas merupakan kesamaan jika dan hanya jika a = b = c. Dari (6)
dan (7) diperoleh
1 3
1 2
 
8 ≥ 1 + 3(abc) 3 + 3(abc) 3 + abc = 1 + (abc) 3 ,
yang ekivalen dengan √
1 3
1 + (abc) 3 ≤ 8 = 2,
atau
1
(abc) 3 ≤ 2 − 1 = 1,
yaitu
abc ≤ 1,
dan kesamaan terjadi jika dan hanya jika a = b = c = 1.

3 Sukubanyak (Polinom)
Misalkan F menyatakan sistem bilangan real atau sistem bilangan rasional dan n adalah bilangan
bulat tidak negatif. Bentuk

f (x) = a0 + a1 x + a2 x2 + · · · + an xn

dengan a0 , a1 , a2 , · · · , an di F dan an 6= 0, disebut sukubanyak atas F berderajad n. Himpunan


semua sukubanyak atas F ditandai dengan F [x]. Berikut diberikan beberapa sifat sukubanyak
yang sering digunakan.

Teorema 2 (Algoritma Pembagian) Misalkan f (x) dan g(x) di F [x] dan g(x) bukan sukubanyak
nol. Maka terdapat sukubanyak q(x) dan r(x) di F [x] yang tunggal dan memenuhi

f (x) = q(x)g(x) + r(x)

dengan r(x) merupakan sukubanyak nol atau r(x) bukan sukubanyak nol yang berderajad kurang
dari derajad g(x).

7
Dalam teorema di atas, q(x) disebut hasilbagi dan r(x) disebut sisa pembagian. Selanjutnya
jika r(x) merupakan sukubanyak nol maka dikatakan f (x) habis dibagi oleh g(x).

Teorema 3 (Teorema Sisa) Jika sukubanyak f (x) dibagi oleh (x − a) maka sisanya adalah f (a).

Bilangan a di F disebut akar dari sukubanyak f (x) jika f (a) = 0. Sebagai akibat dari teorema
di atas diperoleh teorema berikut.

Teorema 4 (Teorema Faktor) Sukubanyak f (x) habis dibagi oleh (x − a) jika dan hanya jika a
merupakan akar dari f (x).

Contoh 6 Sukubanyak f (x) = 2x3 + Ax2 + x − B habis dibagi oleh (x − 2) dan bersisa -9 jika dibagi
(x + 1). Tentukan nilai A dan B.

Penyelesaian: Karena f (x) habis dibagi oleh (x − 2) maka

0 = f (2) = 2(2)3 + A(2)2 + 2 − B = 4A − B + 18. (8)

Karena f (x) bersisa -9 jika dibagi (x + 1) maka

−9 = f (−1) = 2(−1)3 + A(−1)2 + (−1) − B = A − B − 3. (9)

Dengan menyelesaikan sistem persamaan di atas (mengeliminasi (8) dan (9) serta mensubstitusikan
ke salah satu darinya) diperoleh A = −4 dan B = 2.

Contoh 7 Tentukan semua bilangan asli n sehingga sukubanyak x2 + x + 1 membagi sukubanyak


x2n + xn + 1.

Penyelesaian:
Perhatikan bahwa x3 − 1 = (x − 1)(x2 + x + 1) dan x3 − 1 membagi x3m − 1.

(i) Untuk n = 3k,


x2n + xn + 1 = x6k + x3k + 1
= (x6k − 1) + (x3k − 1) + 3
= (x2 + x + 1)Q(x) + 3.

(ii) Untuk n = 3k + 1,
x2n + xn + 1 = x6k+2 + x3k+1 + 1
= x2 (x6k − 1) + x(x3k − 1) + x2 + x + 1
= (x2 + x + 1)R(x).

(iii) Untuk n = 3k + 2,
x2n + xn + 1 = x6k+4 + x3k+2 + 1
= x4 (x6k − 1) + x2 (x3k − 1) + x4 + x2 + 1
= x4 (x6k − 1) + x2 (x3k − 1) + x(x3 − 1) + x2 + x + 1
= (x2 + x + 1)S(x).

8
Jadi x2 + x + 1 membagi x2n + xn + 1 jika dan hanya jika n bukan kelipatan 3.

Sifat yang lain dari sukubanyak yang sering digunakan adalah sifat simetri akar, yang lebih
dikenal dengan nama Teorema Vieta, yaitu hasil tambah dan hasil tambah dari hasil kali akar-akar
suatu sukubanyak.

(a) Jika sukubanyak ax2 + bx + c mempunyai akar-akar x1 dan x2 maka

ax2 + bx + c = a(x − x1 )(x − x2 ) = ax2 − a(x1 + x2 )x + ax1 x2 .

Sehingga
b c
x1 + x2 = − dan x1 x2 = .
a a
(b) Misalkan x1 , x2 , dan x3 akar-akar sukubanyak ax3 + bx2 + cx + d. Dengan ekspansi

a(x − x1 )(x − x2 )(x − x3 ) = ax3 − a(x1 + x2 + x3 )x2 + a(x1 x2 + x2 x3 + x3 x1 )x − ax1 x2 x3

dan komparasi koefisien diperoleh


b c d
x1 + x2 + x3 = − , x1 x2 + x2 x3 + x3 x1 = , dan x1 x2 x3 = − .
a a a

Contoh 8 Misalkan x1 , x2 , dan x3 akar-akar dari x3 + 3x2 − 7x + 1. Tentukan x21 + x22 + x23 .

Penyelesaian: Menurut Teorema Vieta,

x1 + x2 + x3 = −3 dan x1 x2 + x2 x3 + x3 x1 = −7.

Sehingga

9 = (x1 + x2 + x3 )2
= x21 + x22 + x23 + 2 (x1 x2 + x2 x3 + x3 x1 )
= x21 + x22 + x23 + 2(−7).

Oleh karena itu x21 + x22 + x23 = 23.

4 Sistem Persamaan
Bentuk yang melibatkan variabel, yaitu

f (x1, x2 , . . . , xn ) = c

disebut persamaan dengan n buah variabel. Sistem persamaan adalah suatu sistem yang terdiri
dari dua atau lebih persamaan, yaitu

f1 (x1, x2 , . . . , xn ) = c1
f2 (x1, x2 , . . . , xn ) = c2
..
.
fm (x1, x2 , . . . , xn ) = cm

9
Sistem persamaan di atas disebut sistem persamaan dengan n buah variabel dan m persamaan.
Solusi dari suatu sistem persamaan adalah solusi secara simultan dari semua persamaan di dalam
sistem itu. Cara baku untuk mencari solusi suatu sistem persamaan dengan cara eliminasi dan
atau substitusi. Berikut akan diberikan beberapa contoh soal yang tidak regular.

Contoh 9 Cari semua solusi real dari sistem persamaan


2
x+ = 2y
x
2
y+ = 2z
y
2
z+ = 2x.
z
Penyelesaian: Misalkan (x, y, z) solusi sistem persamaan di atas. Diantara x, y, dan z tidak
mungkin ada yang nol. Perhatikan bahwa jika salah satu positif maka dua yang lain juga positif.
Selanjutnya, dengan mengalikan dengan -1 akan diperoleh solusi yang lain. Asumsikan x, y, z > 0.
Dengan menggunakan ketaksamaan AM-GM untuk masing-masing persamaan diperoleh
s  
2 2 √ √
2y = x + ≥ 2 x = 2 2 ⇐⇒ y ≥ 2,
x x
s  
2 2 √ √
2z = y + ≥ 2 y = 2 2 ⇐⇒ z ≥ 2,
y y
s  
2 2 √ √
2x = z + ≥ 2 z = 2 2 ⇐⇒ x ≥ 2.
z z

Dengan menambahkan semua persamaan dari sistem persamaan semula dan hasil di atas, diperoleh
√ √
 
2 2 2
3 2≤x+y+z =2 + + ≤ 3 2.
x y z
√ √ √ √ 
Dengan demikan haruslah x = y = z = 2. Selanjutnya dapat ditunjukkan bahwa 2, 2, 2
√ √ √ 
dan − 2, − 2, − 2 adalah solusi yang dimaksud.

Contoh 10 Cari semua solusi real dari sistem persamaan


4x2
= y
4x2 + 1
4y 2
= z
4y 2 + 1
4z 2
= x.
4z 2 + 1
Penyelesaian: Perhatikan fungsi f : [0, ∞) −→ [0, ∞),

4t2
f (t) = ,
4t2 + 1
merupakan fungsi monoton naik murni.Oleh karena itu jika x < y maka y = f (x) < f (y) = z.
Akibatnya z = f (y) < f (z) = x. Sehingga x < y < z < x, suatu yang tidak mungkin. Dengan cara

10
yang sama jika x > y maka akan diperoleh suatu kontradiksi. Jadi x = y. Dengan menggunakan
argumen yang sama diperoleh y = z. Jadi x = y = z. Dengan menyelesaikan persamaan

4t2
=t
4t2 + 1
diperoleh t = 0 atau t = 21 . Jadi solusi dari sistem persamaan di atas hanyalah tripel (0, 0, 0) dan
1 1 1

2, 2, 2 .

Contoh 11 Tentukan semua solusi real dari sistem persamaan

x2 + y 2 + z 2 = 1


x y + y 2 z 2 + z 2 x2 = 9x2 y 2 z 2
2 2

Penyelesaian: Dari persamaan pertama, tidak mungkin x = y = z = 0. Dari persamaan


kedua tidak mungkin jika satu variabel nol dan dua variabel tidak nol.
Kasus I: Jika dua variabel nol dan satu variabel tidak nol. Misalkan x = y = 0, z 6= 0. Diperoleh
z = ±1. Dengan demikian (0, 0, ±1) merupakan solusi. Dengan cara yang sama diperoleh (0, ±1, 0)
dan (±1, 0, 0) juga merupakan solusi.
Kasus II: Jika ketiga variabel tidak nol. Persamaan kedua ekivalen dengan
1 1 1
2
+ 2 + 2 = 9.
x y z
Digunakan AM-HM, persamaan pertama, dan persamaan di atas diperoleh

1 x2 + y 2 + z 2 3 3 1
= ≥ 1 1 1 = = .
3 3 x2
+ y2
+ z2
9 3

x2 +y 2 +z 2 3
√ √ √
Jadi 3 = 1
+ 12 + 12
. Oleh karena itu x2 = y 2 = z 2 . Diperoleh (± 13 3, ± 13 3, ± 13 3) solusi
x2 y z
sistem persamaan di atas.

Soal-soal Latihan
1. Diketahui a + b = 1 dan a2 + b2 = 2. Tentukan a4 + b4 .
p √ p √
2. Sederhanakan 9 + 80 − 9 − 80 (tanpa melibatkan tanda akar).

3. Buktikan bahwa jika a, b, c ∈ R, dan a2 + b2 + c2 = 1 maka − 12 ≤ ab + bc + ca ≤ 1.

4. Tentukan bilangan real a agar hasil tambah kuadrat akar-akar x2 + (a − 2)x − a − 3 minimum.

5. Hitung nk=1 k!(k 2 + k + 1).


P

6. Tentukan jumlah dari


r r r
1 1 1 1 1 1
1+ 2 + 2 + 1 + 2 + 2 + ··· + 1+ +
1 2 2 3 20012 20022

7. Diketahui f (x) = 2x + 1 dan g(f (x)) = x2 + 3x + 1. Tentukan g(3).

11
8. Misalkan x dan y bilangan real dan x2 + 3xy + y 2 = 60. Tentukan nilai maksimum yang
mungkin untuk xy.

9. Misalkan semua akar dari x6 − 6x5 + ax4 + bx3 + cx2 + dx + 1 = 0 adalah positif. Tentukan
a, b, c, d.

10. Untuk bilangan real a, b, dan c yang memenuhi a ≥ b ≥ c > 0, buktikan bahwa

a2 − b2 c2 − b2 a2 − c2
+ + ≥ 3a − 4b + c.
c a b

11. Buktikan bahwa


1 1 3 5 7 99 1
< . . . ··· < .
15 2 4 6 8 100 10
12. Misalkan a1 , a2 , . . . , an bilangan real positif dan b1 , b2 , . . . , bn adalah penataan kembali dari
a1 , a2 , . . . , an . Buktikan bahwa
a1 a2 an
+ + ··· + ≥ n.
b1 b2 bn

13. Untuk bilangan asli n sebarang, buktikan bahwa


 n+1
1 n 1

(a) 1 + n < 1 + n+1 .
n+1  n+2
(b) 1 + n1 > 1 + n+11
.

14. Buktikan bahwa


1 1 1
√ √ +√ √ + ··· + √ √ > 24.
1+ 3 5+ 7 9997 + 9999
p
n
√ p
n
√ √
15. Buktikan bahwa n+ n
n+ n− n
n < 2 n n untuk n ≥ 2.

16. Misalkan x1 + x2 + x3 = π2 , xi > 0. Buktikan bahwa

1
sin x1 sin x2 sin x3 ≤ .
8

17. Buktikan bahwa tan a + tan b ≥ 2 tan ab untuk setiap a, b ∈ [0, π2 ).

18. Buktikan bahwa untuk bilangan asli n berlaku

n+1 n
 
n! ≤ .
2

19. Misalkan a1 , a2 , . . . , an dan b1 , b2 , . . . , bn adalah bilangan real positif yang memenuhi a1 a2 . . . an =


b1 b2 . . . bn . Buktikan bahwa
(a1 b1 + 1) (a2 b2 + 1) . . . (an bn + 1)
≥ 2n .
b 1 b2 . . . b n

12
20. Cari semua solusi real dari sistem persamaan berikut.

x3 + y = 3x + 4
2y 3 + z = 6y + 6
3z 3 + x = 9z + 8

21. Cari semua tripel (x, y, z) yang memenuhi x4 + y 4 + z 4 − 4xyz = −1.

22. Cari semua solusi real dari sistem persamaan



x+y = 4z − 1

y+z = 4x − 1
p
z+x = 4y − 1

23. Buktikan tidak ada belangan real x, y, x yang memenuhi

x2 + 4yz + 2z = 0
x + 2xy + 2z 2 = 0
2xz + y2 + y + 1 = 0

24. Cari semua bilangan real m sehingga persamaan

(x2 − 2mx − 4(m2 + 1))(x2 − 4x − 2m(m2 + 1)) = 0

mempunyai tepat tiga akar.

Rujukan
[1] Engel, A. 1998. Problem-Solving Strategies. New York: Springer-Verlag.

[2] Larson, L. C. 1983. Problem-Solving Through Problems. New York: Springer-Verlag.

13
TEORI BILANGAN
Nanang Susyanto
Jurusan Matematika FMIPA UGM

1 Sistem Bilangan Bulat

1.1 Latar belakang


Pada zaman dahulu, manusia hanya mengenal sistem bilangan asli

N = {1, 2, 3, ...}

Mereka hanya mengenal sistem bilangan tersebut karena pada waktu itu yang mereka butuhkan
hanyalah menghitung sesuatu yang mereka miliki atau mereka dapatkan. Sebagai contoh: menghi-
tung hasil binatang buruan, menghitung banyak persediaan makanan yangmereka miliki, dan se-
bagainya. Akan tetapi, setelah selang waktu tertentu mereka merasakan binatang buruannya habis
yang kemudian dilambangkan dengan simbol ”0”. Oleh karena itu, mereka mulai mengenal sistem
bilangan cacah
N0 = {0} ∪ N = {0, 1, 2, 3, ...}
Seiring dengan adanya sistem barter, mereka menemui masalah ”jika setiap seekor kambing dapat
ditukar dengan 10 ekor ayam, maka bagaimana jika saya mempunyai 7 ekor ayam yang ingin
ditukarkan dengan seekor kambing?”. Tentu saja orang tersebut masih hutang/kurang 3 ekor
ayam bukan? Kekurangan 3 ekor ini yang kemudian dilambangkan −3. Dari sini mereka mulai
mengenal bilangan bulat
Z = {.., −3, −2, −1, 0, 1, 2, 3, ...}
Perkembangan selanjutnya, jika seseorang mempunyai 1 buah apel sedangkan ia punya dua anak,
maka untuk menuliskan kejadian ini mereka memberi simbol 21 . Dengan demikian, mereka mulai
mengenal sistem bilangan rasional
na o
Q= |a dan b bilangan bulat dengan b 6= 0
b

1.2 Mengingat kembali notasi pada himpunan, relasi serta operasi dua him-
punan
Secara umum, suatu himpunan kita notasikan dengan huruf kapital A, B, C, ... Dalam teori bi-
langan kita hanya akan bekerja pada sistem bilangan asli, cacah, bulat, dan yang paling luas kita
akan bekerja pada sistem bilangan rasional. Untuk himpunan-himpunan tersebut, yaitu himpunan
bilangan asli, cacah, bulat, dan rasional berturut-turut kita notasikan dengan N, N0 , Z, dan Q.
Sedangkan untuk anggota-anggota dari suatu himpunan biasanya kita tulis dengan huruf kecil
a, b, c, ... dan kita akan menggunakan lambang ∈ untuk menyatakan anggota/elemen dan ∈ / untuk
bukan anggota. sebagai contoh: Misalkan A = {1, 2, 3}, maka 1 ∈ A tetapi 5 ∈/ A.

14
1.2.1 Relasi dua himpunan
1. Himpunan A dikatakan himpunan bagian dari B kita tulis dengan A ⊆ B jika untuk setiap
x ∈ A maka x ∈ B.

2. Himpunan A dikatakan sama dengan himpunan B kita tulis dengan A = B jika A ⊆ B dan
B ⊆ A.

3. Himpunan B dikatakan komplemen dari himpunan A kita tulis dengan B = Ac atau B = A0


jika himpunan B berisi semua anggota dari himpunan semesta yang bukan anggota himpunan
A.

1.2.2 Operasi dua himpunan


1. Irisan dua himpunan A dan B dinotasikan dengan A ∩ B adalah himpunan yang anggota-
angotanya merupakan anggota dari kedua himpunan A dan B. Secara matematika dapat
kita tuliskan
A ∩ B = {x|x ∈ A dan x ∈ B}

2. Gabungan dua himpunan A dan B dinotasikan dengan A∪B adalah himpunan yang anggota-
angotanya merupakan anggota himpunan A atau himpunan B. Secara matematika dapat kita
tuliskan
A ∪ B = {x|x ∈ A atau x ∈ B}

3. Selisih dua himpunan A dan B dinotasikan dengan A − B adalah himpunan yang anggota-
anggotanya merupakan anggota himpunan A tetapi bukan anggota himpunan B. Secara
metematika dapat kita tuliskan

A − B = {x|x ∈ A dan x ∈
/ B}

Definisi 1 (sifat tertutup)

Himpunan A dikatakan tertutup terhadap operasi ∗ (bisa penjumlahan, pengurangan, pemba-


gian, perkalian, dan lain-lain) jika untuk setiap a, b ∈ A berlaku a ∗ b ∈ A.
Contoh 1.
Himpunan bilangan bulat tertutup terhadap penjumlahan, pengurangan, dan perkalian, tetapi
tidak terhadap pembagian karena 12 ∈ / Z.

1.3 Himpunan bilangan bulat


Tentu saja himpunan ini telah kita kenal dengan akrab sejak di sekolah dasar. Di sini kita akan
membahas sifat-sifat yang berkaitan dengan himpunan bilangan bulat dan himpunan bagiannya.
Salah satu himpunan bagian dari himpunan bulat adalah himpunan bilangan asli, himpunan ini
beranggotakan bilangan-bilangan bulat yang positif.
Beberapa sifat yang berkaitan dengan bilangan bulat dan himpunan bagiannya

1. Himpunan bilangan bulat tertutup terhadap penjumlahan, pengurangan, dan perkalian,

2. Himpunan bilangan asli tertutup tertutup terhdap penjumlahan dan perkalian,

15
3. Setiap himpunan bagian dari himpunan bilangan asli selalu mempunyai elemen terkecil (min-
imal),

4. Setiap himpunan bagian dari himpunan bilangan bulat yang berhingga selalu mempunyai
elemen minimal dan elemen maksimal,

Sifat (3) disebut sifat terurut rapi (well ordering principle).

1.4 Soal-soal Latihan


1. Tunjukkan hukum D’Morgan yaitu (A ∩ B)c = Ac ∪ B c dan (A ∪ B)c = Ac ∩ B c .

2. Misalkan A ⊆ Z tertutup terhadap pengurangan. Jika diketahui 4 dan 7 merupakan anggota


A,

(a) tunjukkan bahwa 0, 100, 208 ∈ A,


(b) daftarlah semua anggota dari A.

3. Misalkan S adalah himpunan yang memuat semua bilangan bulat. Jika untuk setiap s ∈ S
yang tidak nol, terdapat s0 ∈ S sehingga ss0 = 1, maka tentukan semua anggota-anggota S.

4. Jumlah dari rata-rata aritmatik himpunan A dan rata-rata aritmatik himpunan B adalah
5002. Himpunan A dan B terdiri dari bilangan-bilangan asli berurutan. Jika A ∩ B = {2005}
tentukan kemungkinan unsur himpunan B yang terbesar. (soal olimpiade matematika tk
propinsi tahun 2005)

5. Misalkan S adalah himpunan yang memuat bilangan 1, 2, 3, dan 4. Diketahui untuk sebarang
a, b, c, d ∈ S yang semuanya berbeda akan berlaku ab + cd ∈ S. Selidiki apakah 2008 ∈ S?

6. Buktikan sifat well ordering principle pada sebarang sub himpunan bilangan asli.

7. Suatu barisan bilangan bulat {an } memenuhi persamaan aan +n = an untuk setiap bilangan
asli n. Jika diketahui a2008 = 1, maka tunjukkan bahwa an = 1 untuk setiap bilangan asli n.

2 Teorema Keterbagian

Pada bab ini, kita akan mempelajari tentang konsep dasar keterbagian, algoritma pembagian,
faktor persekutuan terbesar, dan kelipatan persekutuan terkecil.

2.1 Keterbagian

Definisi 1
Misalkan a dan b adalah bilangan bulat dengan a 6= 0. Bilangan a dikatakan habis membagi b
jika terdapat bilangan bulat k sehingga b = ka. Untuk selanjutnya kita tulis a|b, sedangkan dalam
hal a tidak habis membagi b kita tulis dengan a - b.
Contoh 1
3|12 karena terdapat bilangan bulat k yaitu k = 4 sehingga 12 = 4 × 3
3 - 7 karena kita tidak mungkin mendapatkan bilangan bulat k sehinga 7 = k × 3.
Dari definisi di atas kita dapat menurunkan sefat-sifat sebagai berikut

16
Sifat 1
Untuk setiap bilangan bulat a yang tidak nol selalu berlaku a|a dan a|0
Sifat 2
Untuk setiap bilangan bulat a selalu berlaku 1|a
Sifat 3
Jika a|b maka

(i) |a| ≤ |b|

(ii) ac|bc untuk setiap bilangan bulat c yang tidak nol.

Sifat 4
Jika a|b dan a|c maka a| (mb + nc) untuk setiap bilangan bulat m dan n.

Di sini, kita hanya akan membuktikan Sifat 4. Sedangkan untuk sifat-sifat yang lainnya diser-
ahkan kepada pembaca sebagai latihan.
Bukti Sifat 4
Perhatikan bahwa a|b artinya terdapat bilangan bulat k sehingga b = ka, dan juga kita tahu
a|c yang berarti terdapat bilangan bulat l sehingga c = la. Dari kedua fakta tersebut kita punya
mb + nc = mka + nla = (mk + nl) a yang berarti a| (mb + nc) 

Contoh 2
Tentukan semua bilangan asli n sehingga 3n+252n−5 juga merupakan bilangan asli (soal OSN tk
Propinsi, 2002)
Jawab:
Agar 3n+25
2n−5 merupakan bilangan bulat haruslah 2n − 5|3n + 25, di lain pihak kita juga punya
2n − 5|2n − 5. Dengan demikian 2n − 5| (2 (3n + 25) − 3 (2n − 5)) atau ekivalen dengan 2n − 5|65.
Dari sini kita simpulkan 2n−5 = 1, 5, 13, atau 65. Yang selanjutnya kita dapatkan solusi n = 3, 5, 9,
atau 35.

2.2 Algoritma Pembagian

Teorema 1
Jika a dan b adalah bilangan bulat dan b > 0, maka terdapat dengan tunggal bilangan bulat q
dan r sehingga
a = bq + r
dengan 0 ≤ r < b.

Bukti:
Padang himpunan ..., a − 3b, a − 2b, a − b, a, a + b, a + 2b, a + 3b, ... Jika barisan tersebut memuat
unsur nol, maka terdapat bilangan bulat q sehingga a = bq + r dengan r = 0. Jika barisan tersebut
tidak terdapat unsur nol, maka a tidak mungkin nol. Jika a > 0 maka a + ab = a (1 + b) > 0, dan
jika a < 0 maka a − ab = −a (b − 1) > 0. Jadi, barisan tersebut memuat unsur positif. Dengan
demikian, jika kita himpun semua elemen yang positif sebut saja himpunan S, maka menurut well
ordering principle S mempunyai elemen terkecil, sebut elemen minmal tersebut adalah r = a − qb.
Kita akan buktikan bahwa r < b. Jelas r 6= b (mengapa?), andaikan r > b maka akan kita peroleh

17
s = a−(q + 1) b = a−qb−b = r −b > 0. Perhatikan bahwa s ∈ S, dan s < r. Ini kontradiksi dengan
asumsi bahwa r merupakan elemen terkecil. Jadi haruslah terdapat bilangan bulat q sehingga
0 < r = a − bq < b
atau dengan kata lain
a = bq + r dengan 0 < r < b
Sekarang akan kita buktikan ketunggalannya. Misalkan terdapat bilangan bulat 0 ≤ r1 , r2 < b dan
q1 serta q2 sehingga a = bq1 + r1 = bq2 + r2 . Dari sini akan diperoleh b (q1 − q2 ) = r2 − r1 yang
berarti b| (r2 − r1 ) , akan tetapi −b < r2 − r1 < b , akibatnya r2 − r1 = 0 atau dengan kata lain
r2 = r1 . Dengan fakta r2 = r2 ini juga akan berakibat q1 = q2 dan kita selesai.
Dari teorema di atas, dapat kita pahami bahwa jika m suatu bilangan asli, maka untuk sebarang
bilangan bulat n dapat dinyatakan sebagai
n = mk + r
untuk suatu bilangan bulat k dan r dengan 0 ≤ r ≤ m − 1. Bilangan yang berbentuk mk + r
adalah bilangan bulat yang bersisa r ketika dibagi m. Sebagai contoh, kita kita ambil m = 2,
maka fakta di atas mengatakan bahwa setiap bilangan bulat dapat dinyatkan dalam bentuk 2k
atau 2k + 1, yang selanjutnya dalam kehidupan kita sehari-hari bilangan yang berbentuk 2k dan
2k + 1 berturut-turut kita katakan bilangan genap dan bilangan ganjil. Sekarang, marilah kita lihat
beberapa contoh berikut:

Contoh 3
Tentukan semua banyak bilangan asli n dengan n < 2008 yang menyebabkan 31 n (n + 1) meru-
pakan bilangan bulat.
Jawab:
Setiap bilangan asli dapat dinyatakan dalam bentuk 3k, 3k + 1, atau 3k + 2
Untuk n = 3k, kita punya 13 .3k. (3k + 1) = k (3k + 1) merupakan bilangan bulat,
untuk n = 3k + 1 kita punya 13 (3k + 1) (3k + 2) = 3k 2 + 3k + 23 bukan merupakan bilangan
bulat,
untuk n = 3k + 2 kita punya 31 (3k + 2) (3k + 3) = (3k + 2) (k + 1) merupakan bilangan bulat.
Jadi, bilangan asli n yang menyebabkan 13 n (n + 1) bukan bilangan bulat adalah bilangan asli
yang berbentuk 3k + 1. Bilangan seperti ini yang kurang dari 2008 dapat kita daftar dengan cara
berikut:
1 = 3.0 + 1, 4 = 3.1 + 1, 7, ..., 2005 = 3.668 + 1
yang berarti ada 669 bilangan asli kurang dari 2008 yang berbentuk 3k + 1. Dengan ddemikian,
banyak bilangan asli n < 2008 yang menyebabkan 31 n (n + 1) merupakan bilangan bulat adalah
2007 − 669 = 1338.

Contoh 4.
Tunjukkan bahwa tidak ada bilangan kuadrat pada barisan
11, 111, 1111, 11111, ...
Jawab:
Perhatikan bahwa untuk n = 2k kita punya n2 = 4k 2 dan untuk n = 2k + 1 kita punya
n = 4 k 2 + k + 1. Dari sini kita dapat simpulkan bahwa sisa pembagian dari bilangan kadrat
2

oleh 4 adalah 0 atau 1. Sekarang perhatikan barisan


11, 111, 1111, 11111, ...

18
sisa pembagian setiap suku oleh 4 selalu bersisa 3, dengan demikian tidak ada bilangan kuadrat
pada barisan di atas.

2.3 Pembagi sekutu terbesar dan faktor sekutu terkecil


2.3.1 Pembagi sekutu terbesar
Pada saat sekolah dasar, kita semua tentu telah mengenal pembagi sekutu terbesar atau biasa
disebut faktor persekutuan terbesar (FPB), atau disebut juga greatest common divisor (gcd).
Definisi 2.
Diberikan a dan b adalah bilangan bulat yang tidak keduanya nol. Bilangan asli d disebut
pembagi sekutu terbesar dari a dan b atau ditulis dengan d = gcd (a, b) jika

(i). d|a dan d|b

(ii). untuk setiap bilangan asli c dengan c|a dan c|b haruslah berlaku c ≤ d.

bagian (i) mengatakan bahwa d adalah pembagi sekutu dari a dan b, sedangkan bagian (ii)
mengatakan bahwa untuk setiap pembagi sekutu dari a dan b harus lebih kecil atau sama dengan
d, dengan kata lain (ii) mengatakan bahwa d merupakan pembagi sekutu yang terbesar.
Definisi 3.
Bilangan bulat a dan b dikatakan saling prima (relatif prima) jika gcd (a, b) = 1.
Definisi 4.
Untuk sebarang bilangan bulat a, b dan c didefinisikan

gcd (a, b, c) = gcd (gcd (a, b) , c) = gcd (a, gcd (b, c))

Dari definisi di atas, dapat diturunkan beberapa sifat di bawah ini:

1. gcd (a, b) = gcd (b, a) = gcd (|a|, |b|)

2. gcd (a, 1) = 1 untuk setiap bilangan bulat a,

3. gcd (a, 0) = |a| untuk setiap bilangan bulat tak nol a,

4. gcd (ma, mb) = |m| gcd (a, b) untuk setiap bilangan bulat tak nol m,

5. jika d = gcd (a, b) maka gcd ad , db = 1.




Bukti untuk sifat (5) dapat dilihat pada contoh 6, sedangkan untuk yang lain diserahkan kepada
pembaca sebagi latihan.

19
2.3.2 Menentukan gcd dua bilangan dengan algoritma Euclide
Misalkan a dan b bilangan bulat yang tidak keduanya nol. Kita akan menghitung gcd dari a dan b
dengan menggunakan algoritma pembagian yang telah kita kenal pada sub bab sebelumnya. Karena
gcd (a, b) = gcd (b, a) = gcd (|a|, |b|) , maka di sini hanya akan dibahas untuk a dan b bilangan asli
dengan a > b.
Berdasarkan algoritma pembagian, akan terdapat bilangan bulat q dan r dengan 0 ≤ r < b
sehingga a = bq + r atau ekivalen dengan r = a − bq.
Perhatikan bahwa untuk setiap pembagi sekutu a dan b pasti merupakan pembagi dari r. Oleh
sebab itu, dapat kita simpulkan gcd (a, b) = gcd (b, r) . Jika r = 0, maka gcd (a, b) = gcd (b, 0) = b.
Jika r 6= 0 kita dapat lakukan langkah yang sama pada b dan r, yakni terdapat q1 dan r1 dengan
0 ≤ r1 < r sehingga b = rq1 + r1 . Dengan argumen yang sama seperti sebelumnya, kita simpulkan
gcd (b, r) = gcd (r, r1 ) . Jika r1 = 0, maka gcd (b, r) = gcd (r, r1 ) = r. Jika tidak, kita dapat
melakukan langkah di atas sehingga kita peroleh barisan r1 , r2 , ... Akan tetapi, karena a dan b
berhingga, maka tentu akan terdapat n sehingga rn = 0. Dengan demikian

gcd (a, b) = gcd (b, r) = gcd (r, r1 ) = gcd (r1 , r2 ) = ... = gcd (rn−1 , rn ) = gcd (rn−1 , 0) = rn−1

Untuk lebih jelasnya, perhatikan contoh berikut:


Contoh 5.
Hitung gcd (2008, 123456) .
Jawab:
gcd (2008, 123456) = gcd (123456, 2008) . Dengan algoritma pembagian 123456 = 61×2008+968,
dengan demikian gcd (123456, 2008) = gcd (2008, 968) . Kemudian 2008 = 2 × 968 + 72, sehingga
gcd (2008, 968) = gcd (968, 72) , seterusnya dengan algoritma pembagian akan kita peroleh

gcd (968, 72) = gcd (72, 32) = gcd (32, 8) = gcd (8, 0) = 8

dengan demikian gcd (2008, 123456) = 8.


Contoh 6.
Jika d = gcd (a, b) maka tunjukkan bahwa gcd ad , db = 1.


Jawab:
Misalkan gcd ad , db = k, maka kita punya k| ad dan k| db , yang berakibat kd|a dan kd|b dan


selanjutnya kita peroleh kd ≤ d (mengapa?). Dengan demikian, k ≤ 1 dan kita peroleh k = 1


(karena k merupakan bilangan asli). Jadi gcd ad , db = 1.


Teorema 2(Identitas Benzout)


Jika d = gcd (a, b) maka terdapat bilangan bulat x dan y sehingga ax + by = d.
Jawab:
Bentuk himpunan
S = {ax + by|x, y ∈ Z, ax + by > 0}
perhatikan bahwa jika kita ambil x = a dan n = b kita punya a2 + b2 > 0 (ingat pada pendefinisian
gcd kita asumsikan a dan b tidak keduanya nol) yang berarti S tidak kosong. Dengan demikian S
mempunyai elemen terkecil, sebut saja d. Kita akan buktikan bahwa d = gcd (a, b) . Pertama akan
kita buktikan bahwa d|a dan d|b. Dengan algoritma pembagian kita dapat tulis a = dq + r dengan
0 ≤ r < t atau dengan kata lain r = a − dq. Akan tetapi d ∈ S yang berarti d = am + bn untuk
suatu m, n ∈ Z. Oleh karena itu, kita punya r = a − dq = a − (am + bn) q = a − aqm − bnq =
a (1 − qm) − b (nq) ∈ S. Karena d adalah elemen terkecil dari S dan r < t maka r = 0 yang berarti

20
d|a, dengan cara yang sama kita peroleh juga d|b. Sekarang misalkan c adalah sebarang bilangan
asli dengan c|a dan c|b, maka c|am + bn atau c|d. terbukti bahwa d = gcd (a, b) .
Dengan menggunakan teorema di atas kita dapat menurunkan beberapa sifat sebagai berikut:

1. Jika d = gcd (a, b) maka untuk sebarang bilangan bulat c dengan c|a dan c|b haruslah berlaku
c|d.

2. Jika a|bc dan gcd (a, b) = 1, maka a|c.

Bukti:
Untuk sifat (1) dapat langsung dilihat dari pembuktian teorema identitas Benzout, sehingga di
sini hanya akan kita buktikan untuk sifat (2) .
Bukti sifat (2) :
Perhatikan bahwa a|bc, artinya terdapat bilangan bulat k sehingga bc = ka. Selain itu, kita juga
punya gcd (a, b) = 1. Menurut identitas Benzout kita dapat menemukan bilangan bulat x dan y
dengan sifat ax + by = 1. Dengan mengalikan kedua ruas dengan c akan kita peroleh acx + bcy = c
yang ekivalen dengan acx = c − bcy. Perhatikan bahwa a| (c − bcy) , akan tetapi karena a|bcy akan
berakibat a|c.

Contoh 7.
Diberikan gcd (15, 24) = 3. Cari salah satu pasangan bulat (x, y) sehingga 15x + 24y = 3.
Jawab:
Perhatikan bahwa 24 = 15.1 + 9, 15 = 9.1 + 6, dan 9 = 6.1 + 3 (ingat mencari gcd dengan
Algoritma Euclide). Dengan demikian

3 = 9 − 6 = 9 − (15 − 9) = 2.9 − 15 = 2 (24 − 15) − 15 = −3.15 + 2.24

kita dapat mengambil x = −3 dan y = 2.


Contoh 8
Jika gcd (a, b) = 1 dan gcd (a, c) = 1, tunjukkan bahwa gcd (a, bc) = 1.
Jawab:
kita punya gcd (a, b) = 1 dan gcd (a, c) = 1, sehingga ada bilangan bulat x dan y yang memenuhi
ax + by = 1 dan ada bilangan bulat m dan n yang memenuhi ax + cy = 1. Kemudian kita peroleh
(ax + by) (am + cn) = 1 ⇐⇒ a (axm + bmy + cnx) + bc (ny) = 1 yang berakibat gcd (a, bc) |1 dan
tentunya gcd (a, bc) = 1.

Kelipatan persekutuan terkecil. Selain pembagi sekutu terbesar, tentunya pada saat sekolah
dasar juga kita telah mengenal kelipatan persekutuan terkecil (KPK). Dalam pembahasan selan-
jutnya, untuk sebarang bilangan bulat a dan b KPK dari a dan b kita tulis dengan [a, b] .
Definisi 5
Diberikan bilangan bulat a dan b yang tidak keduanya nol. Bilangan bulat positif m disebut
KPK dari a dan b jika

1. m|a dan m|b,

2. untuk setiap bilangan bulat positif n dengan n|a dan n|b haruslah berlaku m ≤ n.

21
Definisi 6
Misalkan a, b, dan c bilangan bulat yang tidak semuanya nol, KPK dari a, b, dan c didefinisikan
sebagai
[a, b, c] = [[a, b] , c] = [a, [b, c]]
Langsung dari definisi di atas, kita dapat menurunkan beberapa sifat sederhana sebagai berikut:

1. [a, b] = [b, a] untuk setiap bilangan bulat a dan b yang tidak keduanya nol,

2. [a, 0] = 0 untuk setiap bilangan bulat tak nol a,

3. [a, 1] = |a| untuk setiap bilangan bulat a.

Teorema 3
Jika a dan b adalah bilangan bulat yang tidak keduanya nol, maka
ab
[a, b] =
gcd (a, b)

Bukti:
Misalkan d = gcd (a, b) , maka a = da1 dan b = db1 untuk suatu bilangan bulat a1 , b1 dengan
gcd (a1 , b1 ) = 1. Misalkan m = da1 b1 . Akan kita buktikan bahwa m = [a, b] . Jelas bahwa a|m dan
b|m. Ambil sebarang bilangan asli n dengan sifat a|n dan b|n, artinya n = ka dan n = lb untuk
suatu bilangan bulat k, l. Dari sini kita dapatkan ka = lb ⇐⇒ kda1 = ldb1 ⇐⇒ ka1 = lb1 . Kita
punya a1 |lb1 , dan karena gcd (a1 , b1 ) = 1 maka a1 |l yang berarti l = ta1 untuk suatu bilangan
bulat t. Dengan demikian kita punya n = lb = ldb1 = tda1 b1 , akibatnya m|n dan kita selesai
membuktikan m = [a, b] .

Contoh 9.
Hitung [56, 72] .
Jawab:
56.72
Karena gcd (56, 72) = 8 maka [56, 72] = 8 = 504.

Contoh 10.
Tentukan bilangan bulat positif terkecil lebih dari 1 yang bersisa 1 ketika dibagi k untuk setiap
2 ≤ k ≤ 10.
Jawab:
Bilangan yang bersisa 1 ketika dibagi k pasti berbentuk km + 1 untuk suatu bilangan bulat m,
dan karena bilangan tersebut harus berbentuk km + 1 untuk setiap 2 ≤ k ≤ 10 maka bilangan
tersebut harus berbentuk rm + 1 dengan r habis dibagi 2 ≤ k ≤ 10. Dengan demikian, r =
[2, 3, 4, ..., 10] = 2520. Jadi bilangan yang dimaksud pasti berbentuk 2520m+1 untuk suatu bilangan
bulat m. Dan karena kita mencari yang terkecil dan lebih besar dari 1 maka kita ambil m = 1.
Jadi, bilangan yang dimaksud adalah 2521.

2.4 Soal-soal Latihan

1. Tentukan semua bilangan bulat p yang menyebabkan


8p+9
(a) 2p+1 merupakan bilangan bulat,

22
(b) 2p + 1 membagi 2p2 + 7,
(c) p2 − 10 kelipatan p + 10.

n3 +24
2. Tentukan semua bilangan asli n sehingga n+3 juga merupakan bilangan asli.

3. Tentukan bilangan asli terbesar n sehingga n3 + 100 kelipatan n + 10.

4. Diberikan f (x) = ax2 + bx + c, dengan a, b, dan c adalah bilangan bulat. Jika 3|f (x) untuk
setiap bilangan bulat x, tunjukkan bahwa 3|a, 3|b, dan 3|c.
n(n+1)(2n+1)
5. Buktikan bahwa 6 merupakan bilangan bulat untuk sebarang bilangan bulat n.

6. Buktikan pernyataan-pernyataan di bawah ini:

(a) hasil kali 2 bilangan bulat yang berurutan selalu habis dibagi 2,
(b) hasil kali 3 bilangan bulat yang berurutan selalu habis dibagi 6,
(c) hasil kali n bilangan bulat yang berurutan selalu habis dibagi n!.

7. Buktikan bahwa gcd (a, b) = gcd (3a + 5b, 11a + 18b) .

8. Buktikan beberapa pernyataan berikut

(a) jika gcd (a, b) dan c|a maka gcd (b, c) = 1


(b) jika gcd (a, b) = 1 dan c| (a + b) maka gcd (a, c) = gcd (b, c) = 1
(c) jika gcd (a, b) = 1 maka gcd a2 , b2 = 1.


n
9. Misalkan an = k 2 , dengan k bilangan asli. Tunjukkan bahwa jika m 6= n maka gcd (am , an ) =
1 untuk k genap, dan gcd (am , an = 2) untuk k ganjil.

10. Jika a, m, n bilangan asli, a > 1 dan gcd (m, n) = d, tunjukkan bahwa

(am − 1) (an − 1)
[am − 1, an − 1] =
ad − 1

(petunjuk: tunjukkan bahwa gcd (am − 1, an − 1) = ad − 1).

3 Bilangan Prima

3.1 Pengertian bilangan prima


Pada pembahasan keterbagian, kita kenal istilah a membagi b. Nah, untuk selanjutnya pernyataan
a membagi b dapat kita katakan a faktor dari b.
Definisi 1.
Bilangan bulat positif p dikatakan bilangan prima jika p mempunyai tepat dua faktor positif
yaitu 1 dan p sendiri.

Definisi 2.
Bilangan bulat positif n dikatakan bilangan komposit jika n mempunyai lebih dari 2 faktor
positif.

23
Definisi 2 di atas juga dapat kita katakan bahwa n adalah bilangan komposit jika terdapat
bilangan bulat positif a, b > 1 sehingga n = ab.
Contoh 1.
Bilangan-bilangan 2, 3, 5, 7, ... merupakan bilangan prima. Bilangan 4, 6, 8, ... merupakan
bilangan komposit.
Teorema 1
Banyak bilangan prima adalah tak hingga.
Bukti:
Andaikan hanya ada sejumlah berhingga bilangan prima, sebut saja p1 , p2 , ..., pn dengan p1 <
p2 < ... < pn . Bentuk N = p1 p2 ...pn + 1, jelas bahwa N > pn . Perhatikan bahwa untuk setiap
k = 1, 2, ..., n haruslah pk tidak membagi N, karena jika pk membagi N maka pk |1 yang jelas tidak
mungkin. Dengan demikian N prima atau terbagi oleh bilangan prima yang lebih dari pn . Hal ini
kontradiksi dengan asumsi kita. Jadi banyak bilangan prima adalah tak hingga.

Contoh 2.
Tentukan semua bilangan prima yang berbentuk n3 + 1 untuk suatu bilangan asli n.
Jawab:
Perhatikan bahwa n3 + 1 = (n + 1) n2 − n + 1 , dengan demikian salah satu faktor yaitu n + 1


atau n2 − n + 1 harus sama dengan 1. Jika n + 1 = 1 maka n = 0 (tidak memenuhi), dan jika
n2 − n + 1 = 1 maka n = 0 atau n = 1. Untuk n = 1 kita peroleh n3 + 1 = 2. Jadi bilangan prima
yang berbentuk n3 + 1 hanyalah 2.

Contoh 3.
Tunjukkan bahwa untuk setiap bilangan prima p selalu berlaku 6| p2 − 1 .


Jawab:
Setiap bilangan prima lebih p > 3 selalu dapat kita nyatakan sebagai 6k + 1 atau 6k − 1. Akan
tetapi, apapun yang terjadi akan selalu kita 2 2
2 2
 peroleh p = 6m + 1 di mana m = 6k + 2k atau
m = 6k − 2k. Dengan demikian 6| p − 1 .

Akibat 1.

1. Jika p prima, maka untuk sebarang bilangan asli n berlaku p|n atau gcd (p, n) = 1.
2. Jika p prima dan p|ab untuk suatu bilangan bulat a dan b, maka p|a atau p|b.

Bukti:

1. Jika p|n, maka tidak ada yang perlu dibuktikan. Asumsikan p - n. Misalkan d = gcd (p, n) ,
yang berarti d|p dan d|n. Karena p prima maka d = 1 atau d = p. Dari asumsi kita punya
p - n, akibatnya d 6= n .Dengan kata lain d = 1 dan kita selesai.
2. Jika p|a, maka kita selesai. Asumsikan p - a. Menurut sifat (1) kita punya gcd (p, a) = 1,
dengan demikian terdapat bilangan bulat x dan y sehingga px + ay = 1. Dengan mengalikan
kedua ruas dengan b akan kita peroleh pbx + aby = b atau setara dengan p (bx) = b − aby.
Akan tetapi p|ab akibatnya p|b.

Akibat 2.
Jika p prima dan n, m sebarang bilangan asli dengan p|nm maka p|n.
Bukti langsung dapat dilihat dari akibat 1 bagian 2.

24
3.2 Faktorisasi Prima
Bilangan prima merupakan bilangan yang lebih sederhana daripada bilangan komposit karena bi-
langan prima hanya mempunyai 2 faktor positif yang berbeda. Oleh karena itu setiap bilangan
asli akan kita bawa ke dalam perkalian bilangan-bilangan prima berpangkat yang disebut dengan
faktorisasi prima. Berikut penjelasannya.

Teorema 2
Setiap bilangan asli n > 1 dapat dinyatakan secara tunggal sebagai

n = pa11 pa22 ...pakk

dengan k suatu bilangan asli, p1 < p2 < ... < pk bilangan-bilangan prima berbeda , dan ai ≥ 1
untuk setiap i = 1, 2, 3, ..., k.

Bukti:
Jika n = p prima, maka kita selesai. Asumsikan n komposit. Misalkan p1 adalah bilangan
prima terkecil yang membagi n, maka n = p1 n1 untuk suatu bilangan asli n1 . Jika n1 prima, maka
kita selesai. Jika n1 tidak prima kita dapat menemukan bilangan prima terkecil yang membagi n1 .
Jika bilangan prima tersebut sama dengan p1 maka n = p21 n2 untuk suatu bilangan asli n2 , jika
bilangan prima tersebut tidak sama dengan p1 sebut saja p2 maka = p1 p2 n2 . Demikian seterusnya,
sampai kita peroleh nm = 1 (mengapa ini dijamin?). Jadi

n = pa11 pa22 ...pakk

Sekarang akan kita buktikan ketunggalannya. Misalkan n = pa11 pa22 ...pakk = q1b1 q2b2 ...qm bm dengan

p1 < p2 < ... < pk dan q1 < q2 < ... < qm bilangan-bilangan prima. Jika terdapat pi yang tidak
sama dengan qt untuk setiap t = 1, 2, ..., m maka pi - q1b1 q2b2 ...qm
bm yang berarti p - n dan ini tidak
i
mungkin. Jadi, untuk setiap pi pasti terdapat t ∈ {1, 2, ..., m} sehingga pi = qt . Dengan cara yang
sama dapat dibuktikan juga bahwa untuk setiap qi pasti terdapat t ∈ {1, 2, ..., k} sehingga qi = pt .
Dengan demikian kita punya k = m. Dan karena p1 < p2 < ... < pk dan q1 < q2 < ... < qk maka
pi = qi untuk setiap i = 1, 2, ..., k atau dengan kata lain

pa11 pa22 ...pakk = pb11 pb22 ...pbkk

Jika terdapat i sehingga ai > bi maka dengan membagi masing-masing ruas dengan pbi i akan be-
rakibat pi membagi ruas kiri tetapi tidak membagi ruas kanan yang jelas tidak mungkin. Demikian
juga jika terdapat i sehingga ai < bi maka dengan membagi kedua ruas dengan pai i akan berakibat
pi tidak membagi ruas kiri tetapi pi membagi ruas kanan yang jelas tidak mungkin. Jadi kita harus
punya ai = bi untuk setiap i = 1, 2, ..., k. Dan kita selesai membuktikan ketunggalannya. 

keterangan: Tidak hanya bilangan asli saja yang dapat kita tulis dalam bentuk perkalian
faktor-faktor prima. Secara umum, untuk sebarang bilangan bulat n 6= 0 selalu dapat ditulis
dalam bentuk n = upa11 pa22 ...pakk dengan u = ±1, k bilangan asli, dan p1 , p2 , ..., pk bilangan prima.

Faktorisasi prima ini, akan memudahkan kita dalam menganalisa suatu bilangan bulat. Per-
hatikan bahwa jika bentuk faktorisasi prima dari n adalah

n = pa11 pa22 ...pakk

maka kita mengetahui beberapa hal sebagai berikut:

25
1. n mempunyai k faktor prima yaitu p1 , p2 , ..., pk .

2. Banyak faktor positif dari n adalah (1 + a1 ) (1 + a2 ) ... (1 + an ) (mengapa?).

Salah satu penggunaan faktorisasi prima adalah mencari FPB dan KPK dua bilangan bulat.
Jika kita sudah mendapatkan faktorisasi prima dari dua bilangan bulat, sebut saja a dan b, maka
pastilah faktorisasi dari keduanya dapat dinyatakan sebagai

a = ps11 ps22 ...pskk dan b = pt11 pt22 ...ptkk

dengan pi prima dan si , ti ≥ 0 untuk i = 1, 2, ..., k. Dengan faktorisasi prima ini, dapat dipahami
bahwa
mk Mk
gcd (a, b) = pm 1 m2 M1 M2
1 p2 ...pk dan [a, b] = p1 p2 ...pk

di mana mi = min {ai , bi } dan Mi =maks{ai , bi } untuk setiap i = 1, 2, ..., k.

Untuk lebih jelasnya, mari kita simak beberapa contoh di bawah ini:

Contoh 4
Tentukan FPB dan KPK dari 56 dan 2008.
Jawab:
Akan kita selesaikan dengan faktorisasi prima, perhatikan bahwa 112 = 24 .7 dan 2008 = 23 .251.
3 4

Dengan demikian gcd (56, 2008) = 2 = 8 dan [112, 208] = 2 (7) (251) = 28 112.
Contoh 5
Tentukan jumlahan dari semua faktor positif dari 5.000.000.
Jawab:
Tentu kita dapat mendaftar semua faktor positif dari 5.000.000 kemudian menjumlahkannya,
namun akan butuh waktu yang sangat lama. Nah, sekarang perhatikan bahwa faktorisasi prima
dari 5.000.000 adalah 26 .57 . Dengan demikian, setiap faktor positifnya berbentuk a b
X2 .5 dengan
2a .5b =

a = 0, 1, 2, ..., 6 dan b = 0, 1, 2, ..., 7. Sehingga jumlahan dari semuanya adalah
X 6  X 7  a,b
a b 7
  38 −1 
2 3 = 2 −1 2 .
a=0 b=0
Contoh 6

Jika m adalah bilangan asli sehingga m merupakan bilangan rasional, maka tunjukkan bahwa
m merupakan kuadrat suatu bilangan asli.
Jawab:

Misalkan m = ab dengan a dan b bilangan asli (mengapa?). Jika b = 1, maka jelas bahwa
m = a2 . Asumsikan b > 1, akibatnya a > 1. Kita dapat tulis faktorisasi prima dari a dan b yaitu a =
2
1 −2t1 2s2 −2t2 k −2sk
ps11 ps22 ...pskk dan b = pt11 pt22 ...ptkk . Sekarang perhatikan bahwa m = ab2 = p2s 1 p2 ...p2s
k =
s1 −t1 s2 −t2 sk −sk  2
p1 p2 ...pk . Karena m bilangan asli dan p1 , p2 , ..., pk bilangan prima maka bilangan
yang berada dalam tanda kurung merupakan bilangan asli, dan kita selesai.

3.3 Soal-soal Latihan

1. Tentukan semua bilangan asli n yang menyebabkan n4 + 4 merupakan bilangan prima.

2. Tunjukkan n merupakan bilangan komposit jika dan hanya jika n terbagi oleh bilangan prima

p dengan p ≤ n.

26
3. Tunjukkan bahwa ada tak hingga bilangan prima yang berbentuk 4n+3 untuk suatu bilangan
asli n.

4. Apakah terdapat bilangan asli n sehingga 6n + 5 merupakan jumlahan dari dua bilangan
prima?

5. Tentukan semua bilangan prima p sehingga 4p2 + 1 dan 6p2 + 1 juga merupakan bilangan
prima. (soal OSN I, Yogyakarta, 2002).

6. Diberikan p > 3 adalah bilangan prima. Jika


1 1 1 a
1+ + + ... + =
2 3 p−1 b

dengan a dan b bilangan bulat dan gcd (a, b) = 1, maka tunjukkan bahwa p membagi a.

7. Tunjukkan bahwa jika a, b, c, d ∈ N dan ab = cd, maka bilangan an + bn + cn + dn merupakan


bilangan komposit untuk setiap bilangan asli n.

8. Jika faktorisasi prima dari n adalah n = pa11 pa22 ...pakk , maka tunjukkan bahwa
m
(a) Hasil kali semua faktor positif dari n adalah n 2 dengan m = (1 + a1 ) (1 + a2 ) ... (1 + ak ) ,
Pa1 i  Pa2 i  Pak i 
(b) Jumlahan semua faktor positif daro n adalah i=0 p1 i=0 p2 ... i=0 pk .

9. Tunjukkan bahwa n merupakan kuadrat sempurna jika dan hanya jika n mempunyai sejumlah
ganjil faktor positif.

10. Terdapat 2008 pintu berjajar diberi nomor 1 sampai 1008 dan semuanya dalam keadaan
tertutup. Sekelompok anak, Pi dengan 1 ≤ i ≤ 2008 berjalan melalui jajaran pintu tersebut.
Masing-masing anak mengubah kondisi pintu nomor k jka dan hanya jika i membagi k, jika
pintunya tertutup diubah menjadi terbuka dan sebaliknya. Cari banyak pintu yang terbuka
setelah semua anak melewati jajaran pintu tersebut.

4 Persamaan dan Sistem Persamaan dalam Bilangan Bulat

Dalam menyelesaikan persamaan dan sistem persamaan dalam bilangan bulat (sering juga disebut
persamaan Diophantine) tentu akan lebih mudah, karena kita hanya dibatasi penyelesaian dalam
bilangan bulat. Sebagai contoh jika kita akan mencari pasangan bilangan real (x, y) yang memenuhi
xy = 2, tentu akan ada tak hingga banyaknya yaitu semua pasangan bilangan real x, x2 untuk


setiap bilangan real tak nol x, pasti merupakan solusi xy = 2. Akan tetapi jika kita akan mencari
pasangan bilangan bulat (x, y) yang memenuhi persamaan xy = 2, maka solusinya hanya ada 4
yaitu (1, 2) , (2, 1) , (−1, −2) , dan (−2, −1) . Mengapa demikian? Untuk lebih jelasnya simak uraian
berikut:

27
4.1 Persamaan Diophantine Linear
Persamaan ini adalah persamaan yang paling sederhana, karena kita bisa langsung mencari solusi
umumnya.

Definisi 1
Misalkan a, b, dan c adalah bilangan-bilangan bulat. Persamaan Diophantine berbentuk ax +
by = c disebut Persamaan Diophantine linear dan setiap pasangan bilangan bulat (x, y) yang
memenuhi ax + by = c disebut solusi.

Teorema 1
Persamaan Diophantine ax + by = c mempunyai solusi jika dan hanya jika gcd (a, b) |c.
Bukti:
=⇒)
Diketahui persamaan ax + by = c mempunyai solusi, artinya ada bilangan bulat x0 dan y0 yang
memenuhi ax0 + by0 = c. Andaikan gcd (a, b) tidak membagi c. Perhatikan bahwa ruas kiri terbagi
oleh gcd (a, b) tetapi ruas kanan tidak terbagi oleh gcd (a, b) yang jelas ini tidak mungkin. Jadi
haruslan gcd (a, b) membagi c.
⇐=)
Diketahui gcd (a, b) |c, artinya terdapat bilangan bulat k sehingga c = k gcd (a, b) .
Menurut identitas Benzout terdapat bilangan bulat m dan n yang memenuhi am + bn =
gcd (a, b) . Dengan mengambil x = km dan y = kn kita akan punya

ax + by = akm + bkn = k (am + bn) = k (gcd (a, b)) = c

yang berarti persamaan ax + by = c mempunyai solusi yaitu (km, kn) .

Contoh 2
Hitung banyak bilangan bulat 1 ≤ n ≤ 100 yang dapat dinyatakan dalam bentuka 6x + 8y
untuk suatu bilangan bulat x dan y.
Jawab:
Perhatikan bahwa gcd (6, 8) = 2. Oleh karena itu menurut teorema di atas, hanya bilangan yang
terbagi oleh 2 yang dapat dinyatakan dalam bentuk 6x + 8y untuk suatu bilangan bulat x dan y.
Dalam hal ini, 1 ≤ n ≤ 100 yang terbagi oleh 2 ada tepat 50 bilangan.

Teorema 2
Jika Persamaan Diophantine ax + by = c mempunyai solusi (x0 , y0 ) maka persamaan tersebut
mempunyai tak hinga banyaknya solusi dan setiap solusinya berbentuk
b a
x (k) = x0 + k dan y (k) = y0 − k
gcd (a, b) gcd (a, b)

untuk sebarang bilangan bulat k.


Bukti:
Diketahui (x0 , y0 ) solusi dari ax + by = c, artinya ax0 + by0 = c. Jika (x (k) , y (k)) kita substi-
tusikan ke persamaan akan kita peroleh
ab ab
ax (k) + by (k) = ax0 + k + by0 − k = ax0 + by0 = c
gcd (a, b) gcd (a, b)

28
yang berarti (x (k) , y (k)) juga merupakan solusi.Nah, untuk bukti bahwa solusi persamaan ax +
by = c hanyalah (x (k) , y (k)) akan kita bahas setelah kita membahas kongruensi bilangan bulat
(pada modul ini tidak dibahas).

Pertanyaannya adalah bagaimana cara kita menentukan solusi awal (x0 , y0 ) ini? Ingat kembali
waktu kita membahas Algoritma Euclide. Kita bisa mencari bilangan bulat m dan n sehingga
am + bn = gcd (a, b) . Karena persamaan ax + by = c punya solusi jika dan hanya jika gcd (a, b) |c
maka terdapat k sehingga c = k gcd (a, b) . Dengan demikian kita bisa mengambil solusi awal
x0 = km dan y0 = kn. (Pada umumnya, bukan pekerjaan yang sulit untuk mencari salah satu
solusi dari persamaan ax + by = c).

Contoh 3
Tentukan semua solusi dari Persamaan Diophantine linear 6x + 8y = 12.
Jawab:
Kita punya gcd (6, 8) = 2 dan 2|12, yang berarti persamaan ini punya solusi. Mudah dipahami
bahwa salah satu solusinya adalah (2, 0) . Dengan demikian solusi umumnya adalah x (k) = 2 + 4k
dan y (k) = −3k untuk sebarang bilangan bulat k.

Contoh 4
Ada berapa banyak pasangan bilangan asli (x, y) yang memenuhi persamaan 4x + 6y = 48?
Jawab:
Untuk menyelesaikan soal ini kita selesaikan seperti biasa yaitu kita cari solusi umumnya, selan-
jutnya kita batasi nilai k agar solusinya merupakan bilangan asli. Salah satu solusi dari persamaan
ini adalah (12, 0) , sehingga solusi umumnya adalah x = 12 + 3k dan y = −2k untuk sebarang
bilangan bulat k. Sekarang akan kita batasi nilai k sehingga x, y > 0. Dari 12 + 3k > 0 kita peroleh
k > −4 dan dari −2k > 0 kita peroleh k < 0, atau dengan kata lain kita peroleh −4 < k < 0.
Akan tetapi, karena k bilangan bulat maka −3 ≤ k ≤ −1. Dengan demikian ada tepat 3 pasangan
bilangan asli (x, y) yang memenuhi 4x + 6y = 48.

4.2 Persamaan Diophantine Non Linear


Persamaan ini sangat banyak bentuknya, kita tidak mungkin mengkarakteristik satu persatu. Di
sini kita hanya memaparkan dengan beberapa teknik melalui contoh-contoh soal:
Contoh 5 (teknik pemfaktoran)
Tentukan solusi bulat dari persamaan xy = 2x − y.
Jawab:
Perhatikan bahwa soal di atas ekivalen dengan xy − 2x + y = 0, dengan menambahkan masing-
masing ruas dengan −2, akan diperoleh xy − 2x + y − 2 = −2 dan ini dapat difaktorkan menjadi
(x + 1) (y − 2) = −2. Karena x dan y bilangan bulat, maka demikian juga dengan x + 1 dan y − 1.
Dengan demikian, ada 4 kejadian yang mungkin

(i). x + 1 = −1 dan y − 2 = 2. Dari sini diperoleh solusi x = −2, y = 4,

(ii). x + 1 = 1 dan y − 2 = −2. Dari sini diperoleh solusi x = 0, y = 0,

(iii). x + 1 = 2 dan y − 2 = −1. Dari sini diperoleh solusi x = 1, y = 1,

(iv). x + 1 = −2 dan y − 2 = 1. Dari sini diperoleh solusi x = −3, y = 3.

29
Mudah dicek bahwa keempat pasang solusi memenuhi persamaan yang diberikan. Jadi semua
solusinya dapat kita nyatakan dalam pasangan (−2, 4) , (0, 0) , (1, 1) , dan (−3, 3) .
Cotoh 6 (teknik pembatasan)
Tentukan bilangan asli a, b, c sehingga a1 + 1b + 1c = 1.
Jawab:
Perhatikan bahwa persamaan di atas simetri, artinya jika a kita tukar dengan b dan b kita
tukar dengan a persamaan tidak berubah. Sehingga dapat kita asumsikan a ≥ b ≥ c. Akibatnnya
1 = a1 + 1b + 1c ≤ 1c + 1c + 1c = 3c ⇔ c ≤ 3. Dari sini kita hanya cukup mengecek untuk c = 1, 2, 3.
1
• c = 1, kita substitusikan ke persamaan awal akan kita peroleh a + 1b = 0, dan ini tidak punya
penyelesaian bilangan asli.
1 1 1
• c = 2, kita substitusikan ke persamaan awal akan kita peroleh a + b = 2
1 1 1 2
Karena a ≥ b, maka kita peroleh 2 = a + b ≤ b ⇔ b ≤ 4. dan juga kita punya b ≥ c = 2.
. • b = 2 tidak ada a yang memenuhi.
. b = 3, kita peroleh a = 6.
. b = 4, kita peroleh a = 4.
1 1
• c = 3, kita substitusikan ke persamaan awal akan kita peroleh a + b = 23 .
Karena a ≥ b, maka kita peroleh 32 = a1 + 1b ≤ 2b ⇔ b ≤ 3. dan juga kita punya b ≥ c = 3
jadi b = 3, sehingga kita peroleh a = 3.
Kita peroleh pasangan solusi (6, 3, 2) , (4, 4, 2) , (3, 3, 3). Perhatikan bahwa awalnya kita asum-
sikan a ≥ b ≥ c, padahal bisa saja a ≥ c ≥ b atau yang lainnya. Tetapi karena persamaan-
nya simetris, maka solusi yang lainnya tingal diubah urutannya. Jadi semua solusinya adalah
(6, 3, 2) , (6, 2, 3) , (3, 2, 6) ,
(3, 6, 2) , (2, 3, 6) , (2, 6, 3) , (4, 4, 2) , (4, 2, 4) , (2, 4, 4) , dan (3, 3, 3) .
Contoh 7 (teknik keterbagian)
Tunjukkan bahwa tidak ada bilangan bulat x dan y yang memenuhi pesamaan
(x + 1)2 + (x + 2)2 + ... + (x + 99)2 = y 2
Jawab:
Andaikan terdapat bilangan bulat x dan y yang demikian. Kita jabarkan yang ruas kiri, yakni:
99.100.199
(x + 1)2 + (x + 2)2 + ... + (x + 99)2 = 99x2 + 99 (100) x + = 99 x2 + 100x + 33.50.199

6
2 2

dengan demikian kita punya 99 x + 100x + 33.50.199 = y . Perhatikan bahwa ruas kiri habis
dibagi 3, akibatnya ruas kanan juga habis dibagi 3. Akan tetapi, karena ruas kanan merupakan
kuadrat sempurna maka ruas kanan juga kan dibagi 9. Tentu saja ruas kiri juga havis dibagi 9,
akibatnya 33.50.199 habis dibagi 9 yang jelas ini tidak mungkin.

Contoh 8 (teknik parameter)


Tunjukkan bahwa persamaan x2 + y 2 = x3 mempunyai tak hingga banyaknya solusi asli.
Jawab:
Persamaan di atas dapat kita tulis sebagai y 2 = x3 − x2 atau ekivalen dengan y 2 = x2 (x − 1) .
Oleh karena itu, agar persamaan tersebut punya solusi, kitaharus punya x − 1 merupakan kuadrat
sempurna. Dengan mengambil x = n2 + 1 dan y = n n2 + 1 untuk sebarang bilangan asli n, maka
2 2
mudah ditunjukkan bahwa pasangan n + 1, n n + 1 merupakan solusi yang banyaknya jelas
ada tak hingga.

30
4.3 Sistem Persamaan Diophantine
Telah kita bahas beberapa jenis dan contoh Persamaan Diophantine serta cara menyelesaikannya.
Nah, untuk menyelesaikan sistem persamaan Diophantine kita dapat membawa ke dalam bentuk
Persamaan Diophantine seperti yang telah kita kenal. Pada dasarnya, kita akan mencari solusi
bilangan asli yang memenuhi semua persamaan yang diberikan secara simultan.

4.4 Soal-soal Latihan

1. Ada berapa banyak pasangan bilangan asli (x, y) yang memenuhi persamaan 2x+3y = 1000?
1 1 1
2. Banyak pasangan bilangan asli (x, y) yang memnuhi persamaan x − y = 3 adalah...
3. Banyaknya bilangan asli n sehingga 3n + 81 merupakan kuadrat sempurna adalah..
4. Bilangan bulat positif terkecil n sehingga 31 membagi 5n + n adalah...
1 1 1
5. Tentukan semua bilangan asli a, b, dan c yang menyebabkan a + b + c merupakan bilangan
asli.
6. Tentukan semua pasangan bilangan bulat non negatif yang memenuhi persamaan
(xy − 7)2 = x2 + y 2

7. Diketahui x, y, z, dan n adalah bilangan-bilangan asli yang memenuhi


xn + y n = z n
Tunjukkan bahwa x, y, dan z semuanya lebih besar dari n.
8. Tentukan semua bilangan real a sehingga persamaan kuadrat x2 + ax + 6a = 0 mempunyai
dua solusi yang keduanya bulat.
9. Tentukan semua pasangan bilangan bulat (x, y) yang memenuhi
x4 + x3 + x2 + x + 1 = y 2

10. Carilah semua bilangan prima p sehingga sistem persamaan


p + 1 = 2x2


p2 + 1 = 2y 2
mempunyai solusi bulat.

5 Fungsi Tangga

Dalam dunia jual beli, biasanya penjual ingin menjual harganya semahal mungkin, dan sebaliknya
pembeli ingin membeli barang yang ia inginkan semurah mungkin. Bahkah, kadang-kadang jika
harganya tidak bulat ribuan misalnya 7.300 maka penjual ingin dibayar 7.500, sebaliknya pembeli
ingin membayar dengan harga yang dibulatkan ke bawah yaitu 7.000. Nah, dalam ilmu matematika
kita akan mengenal fungsi yang digunakan oleh penjual dan pembeli di atas. Fungsi yang akan
kita pelajari jika digambarkan pada bidang kartesius akan berbentuk seperti tangga. Ada 3 macam
fungsi tangga yang akan kita bahas dalam bab ini, yaitu fungsi floor (pembulatan ke bawah), fungsi
ceiling (pembulatan ke atas), dan fungsi bulat (pembulatan ke bilangan bulat yang terdekat).

31
5.1 Fungsi floor
Fungsi floor disebut juga fungsi pembulatan ke bawah, yakni dengan mengambil bagian bulatnya.
Untuk sebarang bilangan real x,nilai fungsi floor dari x kita tulis dengan bxc .
Definisi 1
Misalkan x adalah sebarang bilangan real. Nilai fungsi floor x kita tulis dengan bxc merupakan
bilangan bulat terbesar yang kurang dari atau sama dengan x.
Contoh: √ 
b3, 14c = 3, b−2, 5c = −3, 2 = 1, dan lain sebagainya.

Definisi 2
Untuk sebarang bilangan real x, notasi {x} menyatakan bagian pecahan dari x.
Secara matematika, definisi di atas dapat kita tuliskan
{x} = x − bxc
Dari sini jelas bahwa untuk sebarang bilangan real x berlaku 0 ≤ {x} < 1.
Contoh: √
{3, 14} = 0, 14; {−2, 5} = 0, 5; 2 = 0, 41..., dan lain sebagainya.

Langsung dari definisi, kita dapat menurunkan beberapa sifat sebagai berikut:
1. Untuk sebarang bilangan real x selalu berlaku x − 1 < bxc ≤ x,
2. bxc = x jika dan hanya jika x ∈ Z,
3. bx + kc = bxc + k untuk sebarang bilangan bulat k,
4. bxc + byc ≤ bx + yc untuk setiap x, y ∈ R,
5. bxyc ≤ bxc byc untuk setiap x, y ∈ R.
Sifat 1, 2, dan 3 trivial. Di sini kita hanya akan membuktikan sifat 4, sedangkan untuk sifat 5
buktinya hampir sama dengan pembuktian sifat 4 dan diserahkan kepada pembaca sebagai latihan.
Bukti sifat 4.
Tulis x = bxc + {x} dan y = byc + {y} , akan kita peroleh
bx + yc = bbxc + byc + {x} + {y}c = bxc + byc + b{x} + {y}c ≥ bxc + byc
dan kita selesai. 

Contoh 1
Untuk sebarang bilangan real x, tunjukkan bahwa x + 21 + bxc = b2xc .
 

Bukti:
1
Tulis x = bxc + {x} dengan 0 ≤ {x} < 1. Kita bagi 2 kasus, yaitu jika 0 ≤ {x} < 2 dan
1
2 ≤ {x} < 1.
(i). untuk 0 ≤ {x} < 12 , kita punya
     
1 1 1
x+ + bxc = bxc + {x} + + bxc = 2 bxc + {x} + = 2 bxc
2 2 2
dan
b2xc = b2 bxc + 2 {x}c = 2 bxc + b2 {x}c = 2 bxc
yang jelas bahwa ruas kiri sama dengan ruas kanan.

32
1
(ii). untuk 2 ≤ {x} < 1, kita punya
     
1 1 1
x+ + bxc = bxc + {x} + + bxc = 2 bxc + {x} + = 2 bxc + 1
2 2 2

dan
b2xc = b2 bxc + 2 {x}c = 2 bxc + b2 {x}c = 2 bxc + 1
yang juga jelas bahwa ruas kiri sama dengan ruas kanan.

Perhatikan bahwa jika n dan a adalah sebarang bilangan asli, mudah dipahami bahwa bn/ac
merupakan banyaknya bilangan kelipatan a di antara 1, 2, ..., n. Fakta ini sederhana tetapi dapat
kita gunakan untuk menyelesaikan beberapa permasalahan seperti pada contoh berikut.
Contoh 2
Tentukan banyak anggota himpunan {1, 2, ..., 100} yang habis dibagi 2 atau 3.
Jawab:
Banyak bilangan kelipatan 2 ada b100/2c = 50, banyak bilangan kelipatan 3 ada b100/3c = 33,
dan banyak bilangan kelipatan 2 dan 3 ada b100/6c = 16. Oleh karena itu, dengan prinsip inklusi
eksklusi kita peroleh bahwa banyak kelipatan 2 atau 3 ada b100/2c + b100/3c − b100/6c = 50 +
33 − 16 = 67 bilangan.
Contoh 3
Hitung banyak nol di sebelah kanan tanpa terputus dari 31!.
Jawab:
Angka nol di sebelah kanan tanpa terputus pada 31! akan dihasilkan pada saat kita menga-
likankelipatan 10, dan faktor 10 ini didapat dari kelipatan genap dan kelipatan 5. Karena banyak
kelipatan 2 lebih banyak daripada banyak kelipatan 5, maka kita cukup menghitung kelipatan 5.
Perhatikan juga bahwa mungkin bilangan kelipatan 5 yang kita kalikan tersebut juga merupakan
kelipatan 25 dan jika dikalikan dengan bilangan kelipatan 4 akan menghasilkan bilangan kelipatan
100 yang akan menambah nol di sebelah kanan sebanyak 2, akan tetapi sebanyak 1 nol telah telah
kita masukkan saat kita menghitung kelipatan 5. Dengan demikian, banyak nol di sebelah kanan
tanpa terputus dari 31! adalah b31/5c + b31/25c = 6 + 1 = 7 nol.

5.2 Fungsi ceiling


Fungsi floor disebut juga fungsi pembulatan ke atas. Untuk sebarang bilangan real x,nilai fungsi
ceiling dari x kita tulis dengan dxe .
Definisi 3
Misalkan x adalah sebarang bilangan real. Nilai fungsi ceiling x kita tulis dengan dxe merupakan
bilangan bulat terkecil yang lebih dari atau sama dengan x.
Contoh: √ 
d3, 14e = 4, d−2, 5e = −2, 2 = 2, dan lain sebagainya.

5.3 Fungsi bulat


Fungsibulat disebut juga fungsi pembulatan ke bilangan bulat terdekat. Untuk sebarang bilangan
real x,nilai fungsi bulat dari x kita tulis dengan [x] .
Definisi 4

33
Misalkan x adalah sebarang bilangan real. Nilai fungsi bulat x kita tulis dengan [x] merupakan
bilangan bulat terdekat dengan x. Jika x = k+ 21 untuk suatu bilangan bulat k, maka kita definisikan
[x] = k + 1.
Contoh: √ 
[3, 14] = 3, [−2, 5] = −2, 2 = 1, dan lain sebagainya

Dari definisi di atas, kita dapat menurunkan beberapa sifat berikut:

1. bxc ≤ [x] ≤ dxe untuk setiap bilangan real x.

2. dxe = x jika dan hanya jika [x] = x,

3. dx + ke = dxe + k dan [x + k] = [x] + k untuk sebarang bilangan bulat k,

4. dxe + dye ≥ dx + ye untuk setiap x, y ∈ R,

5. dxye ≤ dxe dye untuk setiap x, y ∈ R.

Contoh 4
Tunjukkan bahwa untuk setiap bilangan real x selalu berlaku x + 12 = [x]
 

Jawab:
0 ≤ {x} < 1. Jika 0 ≤ {x} < 21 maka x + 12 = bxc dan [x] = bxc .
 
Tulis x = bxc + {x} ,dengan
Jika 12 ≤ {x} < 1 maka x + 12 = bxc + 1 dan [x] = bxc + 1.


Contoh 5
Tentukan semua bilangan real x yang memenuhi persamaan
 2  2
x + x = 2003

(soal OSN SMA tahun 2003, bidang matematika)


Jawab:
2003
Perhatikan bahwa jika x2 bulat maka x2 = x2 yang akan berakibat 2
   
2
 2  x =2  2 yang jelas
bukan
 2  bilangan bulat. Jadi, x harus bukan bilangan
 2bulat.
 Akibatnya x = x + 1, sehingga
2 x + 1 = 2003 yang selanjutnya kita peroleh x = 1001. Dari sini kita simpulkan 1001 ≤

2 < 1002 yang kemudian kita dapatkan penyelesaiannya adalah − 1002 < x ≤ − 1001 atau

x
√ √
1001 ≤ x < 1002.

5.4 Soal-soal Latihan


1. Buktikan atau beri contoh penyangkal dari pernyataan-pernyataan berikut

(a) [x + y] ≤ [x] + [y] untuk setiap x, y ∈ R,


(b) [xy] ≤ [x] [y] untuk setiap x, y ∈ R,
j k  
(c) bxc
m = m x
untuk setiap x ∈ R dan bilangan bulat m.

2. Misalkan x dan y adalah bilangan real yang memenuhi bx + yc = bxc + byc dan b−x − yc =
b−xc + b−yc . Buktikan bahwa x atau y merupakan bilangan bulat.

34
3. Untuk setiap bilangan real x didefinisikan bxc sebagai bilangan bulat terbesar yang kurang
dari atau sama dengan x. Misalkan a dan b adalah bilangan real positif sehingga a bac = 17
dan b bbc = 11. Tentukan nilai dari a − b.

4. Hitung
1 1 1 1
+ + + ... +
[1] [2] [3] [1000]

5. Cari semua bilangan asli n sehingga banyak nol di sebelah kanan tanpa terputus dari n! tepat
ada 10 nol.

6. Diketahui S = {1, 2, ..., 100} . Hitung banyaknya anggota S yang merupakan kelipatan 2 atau
kelipatan 3 tetapi bukan kelipatan 5.

7. Tentukan semua bilangan real x yang memenuhi persamaan bxc dxe = x2

8. Hitung banyak bilangan real x dengan 1 ≤ x ≤ 100 yang memenuhi persamaan

x2 − x2 = (x − bxc)2
 

2
(m+2)2
j 2k j k j k
9. Tunjukkan bahwa untuk setiap m bilangan asli, m3 + (m+1) 3 + 3 merupakan
kuadrat sempurna.
j √ n k
10. Misalkan an = 1 + 2 . Tunjukkan bahwa an ganjil jika n genap, dan genap jika n
ganjil.

35
Geometry
Fajar Yuliawan

Sebelum kita mulai pembahasan bab geometri ini, ada beberapa hal yang perlu
diketahui oleh pembaca. Geometri yang dibahas di sini tidak akan mencakup se-
bagaian besar geometri dasar yang pernah diajarkan di sekolah menengah. Hal-hal
dasar yang tidak dibahas di sini adalah hubungan antara garis dengan titik, hubun-
gan antara garis dengan garis, kesebangunan dan kekongruenan segitiga, hubungan
garis dengan lingkaran (berpotongan dan bersinggungan), pengenalan trigonometri
dan bangun-bangun ruang sederhana. Materi-materi tersebut dapat didapatkan
sendiri di sebagian besar buku-buku matematika untuk sekolah menengah.

Pembahasan geometri disini selanjutnya lebih ditekankan pada pembuktian-


pembuktian beberapa teorema maupun fakta-fakta dalam geometri. Teorema
yang dibahas di bab geometri ini memang tidak banyak, namun diharapkan dapat
mengenalkan pembaca pada pembuktian fakta-fakta geometri dan menjadi dasar
pembuktian-pembuktian soal-soal olimpiade. Seringkali untuk membuktikan se-
buah soal olimpiade, yang diperlukan adalah ide-ide pada pembuktian suatu teo-
rema, bukan teorema yang bersangkutan. Oleh karena itu, pembaca diharapkan
dapat memahami setiap bukti yang ada pada setiap teorema. Dalam hal ini,
pembaca diharapkan dapat mengetahui dua hal, yaitu kebenaran setiap langkah
pembuktian dan tujuan setiap langkah tersebut. Jika ada keraguan mengenai su-
atu langkah pembuktian, pembaca seharusnya menanyakan hal ini kepada teman
maupun guru matematika di sekolah.

1 Titik-titik dan Garis-garis yang Terhubung de-


ngan Segitiga

Pertama, kita buat beberapa konvensi ntuk mempermudah penulisan. Untuk se-
tiap segitiga ABC, kita gunakan notasi untuk panjang, yaitu a = BC, b = CA,
c = AB, s = (a + b + c)/2 dan notasi untuk sudut, yaitu A = ∠BAC, B = ∠ABC,
dan C = ∠BCA. Kemudian untuk luas, kita menggunakan notasi [XY Z] untuk
menyatakan luas segitiga XY Z. Lebih umum, untuk setiap poligon P , maka [P ]
menyatakan luas poligon tersebut.

36
1.1 Konkurensi dan Kolinearitas pada Segitiga: Teorema
Ceva dan Teorema Menelaos

Kita mulai dengan segmen-segmen garis yang menghubungkan titik sudut suatu
segitiga dengan sebuah titik yang terletak pada sisi di depan titik sudut tersebut.
Segmen garis seperti itu disebut sebagai cevian (diambil dari nama Giovanni Ceva,
seorang matematikawan Italia yang pertama kali menyinggung masalah konkurensi
tiga buah cevian).

Teorema Ceva. Misalkan ABC sebuah segitiga dan D, E, F tiga titik yang
berturut-turut terletak pada sisi-sisi BC, CA, AB. Maka garis-garis AD, BE, CF
konkuren jika dan hanya jika
BD CE AF
= 1.
DC EA F B
Bukti. Teorema di atas membutuhkan pembuktian ”dua arah”, yaitu: jika
AD, BE, CF konkuren, maka kesamaan di atas berlaku dan jika kesamaan berlaku,
maka AD, BE, CF konkuren.
A A

F E F' E
F
P P'

B C C
D B D

Gambar 1.

Pertama, kita buktikan dulu bahwa jika AD, BE, CF konkuren maka ke-
samaan yang diberikan berlaku. Misalkan P adalah titik perpotongan ketiga garis
AD, BE, CF . Perhatikan dua identitas berikut:
BD [ABD] BD [P BD]
= dan = ,
DC [ACD] DC [P CD]
yang diperoleh dari fakta bahwa jika dua buah segitiga memiliki ”tinggi” yang
sama, maka perbandingan luasnya sama dengan perbandingan ”alas”-nya. Dari
dua identitas tersebut, kemudian kita peroleh
BD [ABD] − [P BD] [AP B]
= = (Mengapa?).
DC [ACD] − [P CD] [CP A]

37
Dengan cara yang sama, kita peroleh

CE [BP C] AF [CP A]
= dan = .
EA [AP B] FB [BP C]

Jadi,
BD CE AF [AP B] [BP C] [CP A]
= = 1.
DC EA F B [CP A] [AP B] [BP C]

Sekarang misalkan kesamaan di atas berlaku. Akan dibuktikan bahwa AD,


BE, CF berpotongan di satu titik. Untuk membuktikan hal ini, kita meng-
gunakan teknik titik bayangan (phantom point). Perhatikan Gambar 1 sebelah
kanan. Misalkan cevian AD dan BE berpotongan di titik P 0 dan garis CP 0 mem-
otong sisi AB di titik F 0 . Kita cukup membuktikan bahwa F 0 = F , atau dengan
kata lain, kedua titik tersebut berimpit (Mengapa?). Untuk membuktikan hal ini,
pertama perhatikan bahwa tiga cevian AD, BE, CF 0 konkuren (bertemu di titik
F 0 ). Dengan demikian, kita punya

BD CE AF 0 BD CE AF
0
=1= ,
DC EA F B DC EA F B
sehingga
AF 0 AF
0
= .
FB FB
Dari sini kita simpulkan F = F 0 (Mengapa?) dan kita selesai.

Pada kasus-kasus tertentu, teorema Ceva di atas lebih mudah digunakan dalam
bentuk trigonometri berikut:

Akibat (”Trig Ceva”). Misalkan ABC sebuah segitiga dan P, Q, R tiga


titik yang berturut-turut terletak pada sisi-sisi BC, CA, AB. Maka garis-garis
AP, BQ, CR konkuren jika dan hanya jika
sin ∠CAP sin ∠ABQ sin ∠BCR
= 1.
sin ∠AP B sin ∠QBC sin ∠RCA

Akibat di atas dapat dibuktikan dengan mudah dengan menggunakan aturan


sinus dan teorema Ceva atau secara langsung dengan menggunakan aturan sinus
pada beberapa segitiga. Bukti selengkapnya diserahkan kepada pembaca

Tiga buah titik dikatakan kolinear jika terletak pada satu garis. Kriteria kolin-
earitas tiga titik yang berada pada ketiga sisi-sisi segitiga diberikan oleh Menelaos.

38
Teorema Menelaos. Misalkan ABC sebuah segitiga dan D, E, F tiga titik
pada garis-garis BC, CA, AB (D, E, F bisa terletak pada perpanjangan sisi-sisi
segitiga ABC). Maka D, E, F kolinear jika dan hanya jika

BD CE AF
= 1.
DC EA F B
Bukti. Pertama, kita buktikan bahwa jika D, E, F kolinear, maka kesamaan
yang diberikan berlaku. Buat garis tegak lurus dari tiga A, B, C terhadap garis
yang melalui D, E, F dan misalkan P , Q, R adalah ketiga kaki tegaklurusnya
(perhatikan Gambar 2).
A

Q
F

P
E
R

B C D

Gambar 2.

Kita punya tiga kesamaan berikut yang dapat diperoleh dengan meninjau
kesebangunan-kesebangunan beberapa segitiga:
BD BQ CE CR AF AP
= , = , dan = .
DC CR EA AP FB BQ
Dengan mengalikan ketiga kesamaan tersebut, kita peroleh
BD CE AF BQ CR AP
= = 1.
DC EA F B CR AP BQ

Bukti untuk arah yang satunya (yaitu jika kesamaan berlaku, maka ketiga
titik kolinear) dapat dibuktikan dengan menggunakan titik bayangan, sama seperti
bukti teorema Ceva. Hal ini dilakukan dengan memisalkan F 0 sebagai perpotongan
garis-garis AB dan DE lalu membuktikan bahwa F = F 0 . Bukti selengkapnya
diserahkan kepada pembaca.

39
1.2 Panjang Cevian: Teorema Stewart

Panjang cevian dapat dihitung dengan menggunakan teorema berikut:


Teorema Stewart. Misalkan AX adalah sebuah cevian dengan panjang p
yang membagi sisi BC menjadi dua segmen, yaitu BX dengan panjang m dan
XC dengan panjang n. Maka berlaku
a(p2 + mn) = b2 m + c2 n.
Bukti.
A

c p b

B m X n C

Gambar 3.
Dengan menggunakan aturan cosinus pada segitiga ABX dan ACX, kita per-
oleh
p2 + m2 − c2 p 2 + n 2 − b2
cos ∠AXB = dan cos ∠AXC = .
2pm 2pn
Karena ∠AXB = 180◦ − ∠AXC, maka cos ∠AXB = − cos ∠AXC atau setara
dengan cos ∠AXB + cos ∠AXC = 0. Dengan demikian, kita punya
p 2 + m 2 − c 2 p 2 + n 2 − b2
+ = 0,
2pm 2pn
yang setara dengan
n(p2 + m2 − c2 ) + m(p2 + n2 − b2 ) = 0,
atau setara juga dengan
(m + n)(p2 + mn) = b2 m + c2 n
dan setara dengan kesamaan yang diinginkan, karena m + n = a.
Dengan teorema di atas, panjang cevian AX dapat dihitung secara langsung,
yaitu r
b2 m + c 2 n
p= − mn.
a

40
1.3 Titik-titik dan Garis-garis Istimewa pada Segitiga

Pada sebuah segitiga, terdapat banyak titik dan garis istimewa, namun dalam
subbab ini, hanya akan dijelaskan beberapa diantaranya, yaitu garis berat, titik
berat, garis tinggi, titik tinggi, garis bagi sudut, titik pusat lingkaran dalam, garis
sumbu dan titik pusat lingkaran luar.

1.3.1 Garis Berat (Median) dan Titik Berat (Centroid)

Kita mulai dengan definisi garis berat. Garis berat sebuah segitiga adalah garis
yang melalui titik sudut segitiga tersebut dan titik tengah sisi di depannya. Dengan
demikian, setiap segitiga akan memiliki tiga garis berat. Dengan menggunakan
teorema Ceva, teorema berikut dapat dibuktikan dengan mudah:

Teorema. Misalkan ABC sebuah segitiga dan A0 , B 0 , C 0 titik-titik tengah sisi-


sisi BC, CA, AB. Maka garis-garis berat AA0 , BB 0 , CC 0 konkuren.
A

C' B'
G

B C
A'

Gambar 4.

Titik potong ketiga garis berat sebuah segitiga kemudian dinamakan titik berat.
Karena berpotongan di satu titik, maka ketiga garis berat sebuah segitiga akan
membagi segitiga tersebut menjadi enam bagian. Selain itu, setiap garis berat akan
membagi garis berat lainnya menjadi dua bagian. Kita punya teorema berikut:

Teorema. Misalkan AA0 , BB 0 , CC 0 adalah tiga garis berat segitiga ABC yang
berpotongan di titik berat G. Maka keenam segitiga AGB 0 , AGC 0 , BGA0 , BGC 0 ,
CGA0 dan CGB 0 memiliki luas yang sama.
Bukti. Perhatikan kembali Gambar 4. Karena A0 , B 0 , C 0 berturut-turut
adalah titik-titik tengah BC, CA, AB, kita punya bahwa [BGA0 ] = [CGA0 ],

41
[AGB 0 ] = [CGB 0 ], dan [AGC 0 ] = [BGC 0 ]. Kita juga punya bahwa [ABA0 ] =
[ACA0 ], sehingga

2[AGC 0 ] = [ABG] = [ABA0 ] − [BGA0 ]


= [ACA0 ] − [CGA0 ] = [ACG]
= 2[AGB 0 ],

sehingga [AGC 0 ] = [AGB 0 ]. Dengan cara yang sama diperoleh [BGA0 ] = [AGB 0 ] =
[AGC 0 ], dan kesimpulan mengikuti.

Teorema. Misalkan AA0 , BB 0 , CC 0 adalah tiga garis berat segitiga ABC yang
berpotongan di titik berat G. Maka
AG BG CG
0
= 0
= = 2.
GA GB GC 0
Bukti. Kita gunakan lagi teorema sebelumnya. Kita punya bahwa

AG [ABG] [AGC 0 ] + [BGC 0 ]


= = = 2,
GA0 [GBA0 ] [GBA0 ]

karena [AGC 0 ] = [BGC 0 ] = [GBA0 ]. Kesamaan lain dapat diperoleh dengan cara
yang sama.

Panjang garis berat sendiri dapat dihitung dengan mudah menggunakan teo-
rema Stewart.

1.3.2 Garis Bagi Sudut (Bisector)

Garis yang membagi sebuah sudut segitiga menjadi dua bagian yang sama besar
dinamakan garis bagi sudut. Pertama, kita punya teorema berikut:

Teorema. Misalkan AA0 , BB 0 , CC 0 adalah garis-garis bagi sudut segitiga ABC


(dengan demikian, ∠A0 AB = ∠A0 AC = 12 A, dan seterusnya). Maka

A0 B c B0C a C 0A b
0
= , 0
= , dan 0
= .
AC b BA c CB a
Bukti.

42
A

B A' C

Gambar 5.

Dengan aturan sinus pada segitiga-segitiga AA0 B dan AA0 C, kita peroleh
A0 B sin ∠A0 AB A0 C sin ∠A0 AC
= dan = .
AB sin ∠AA0 B AC sin ∠AA0 C
Karena ∠A0 AB = ∠A0 AC dan sin ∠AA0 B = 180◦ −sin ∠AA0 C, maka sin ∠A0 AB =
sin ∠A0 AC dan sin ∠AA0 B = sin ∠AA0 C. Akibatnya,
A0 B sin ∠A0 AB sin ∠A0 AC A0 C
= = = ,
AB sin ∠AA0 B sin ∠AA0 C AC
sehingga kita peroleh
A0 B AB c
0
= = .
AC AC b
Dua kesamaan sisanya dapat dibuktikan dengan cara yang sama.

Dengan menggunakan teorema di atas dan teorema Ceva atau langsung dengan
menggunakan Trig Ceva, kita peroleh teorema berikut:

Teorema. Misalkan AA0 , BB 0 , CC 0 adalah tiga garis bagi sudut segitiga ABC.
Maka ketiga garis tersebut konkuren.

Selain itu, karena perbandingan A0 B/A0 C dan A0 B + A0 C dapat dinyatakan


dalam panjang sisi-sisi a, b, c, maka panjang A0 B dan A0 C juga dapat dinyatakan
dalam a, b, c. Selanjutnya, dengan teorema Stewart, panjang garis bagi sudut AA0
juga dapat dihitung. Perhitungan ini diserahkan kepada pembaca sebagai latihan.

1.3.3 Garis Tinggi (Altitude) dan Titik Tinggi (Orthocenter)

Garis tinggi sebuah segitiga adalah garis yang melalui titik sudut sebuah segitiga
dan tegak lurus dengan sisi di depannya. Kita punya teorema berikut

43
Teorema. Misalkan AA0 , BB 0 , CC 0 garis-garis tinggi sebuah segitiga (dengan
demikian, AA0 tegak lurus BC, dan seterusnya). Maka ketiga garis tersebut
konkuren.
Bukti.
A

C'

B'

B A' C

Gambar 6.

Kasus dimana ABC segitiga siku-siku trivial (sangat jelas dan tidak ada yang
perlu dibuktikan), karena ketiga garis tinggi akan berpotongan pada titik sudut
siku-siku segitiga ABC). Jadi kita cukup meninjau dimana ABC bukan segitiga
siku-siku.

Sekarang akan kita buktikan teorema tersebut untuk kasus dimana ABC se-
gitiga lancip. Kita punya bahwa
AA0 AA0
= tan B dan = tan C.
BA0 CA0
Dengan demikian,
BA0 tan C
0
= .
CA tan B
Dengan cara yang sama, diperoleh
CB 0 tan A AC 0 tan B
0
= dan 0
= .
AB tan C BC tan A
Akibatnya,
BA0 CB 0 AC 0 tan C tan A tan B
= = 1,
A0 C B 0 A C 0 B tan B tan C tan A
sehingga AA0 , BB 0 , CC 0 konkuren.

Kasus terakhir dimana segitiga ABC tumpul diserahkan kepada pembaca. Satu
hal yang perlu diperhatikan dalam pembuktian kasus tersebut adalah bahwa titik
perpotongan ketiga garis tinggi terletak di luar segitiga ABC.

44
Titik potong ketiga ketiga garis tinggi sebuah segitiga selanjutnya disebut se-
bagai titik tinggi (orthocenter) segitiga. Kemudian, sama seperti pada garis bagi
sudut, panjang BA0 dan CA0 dapat dinyatakan dalam panjang sisi-sisi a, b, c dan
fungsi trigonometri sudut-sudut A, B, C. Dengan menyatakan fungsi trigonometri
sudut dalam panjang sisi (misalnya dengan aturan cosinus), panjang BA0 dan CA0
dapat dinyatakan dalam panjang sisi-sisi a, b, c. Selanjutnya, teorema Stewart
dapat digunakan untuk menghitung panjang garis tinggi AA0 .

1.3.4 Garis Sumbu (Perpendicular Bisector)

Garis sumbu sebuah segitiga adalah garis yang melalui titik tengah sebuah sisi dan
tegak lurus terhadap sisi tersebut. Kita punya teorema berikut:

Teorema. Misalkan lA , lB , lC adalah garis-garis sumbu segitiga ABC yang


berturut-turut tegak lurus terhadap sisi-sisi BC, CA, AB (dengan demikian, lA
melalui titik tengan BC, dan seterusnya). Maka lA , lB , lC konkuren.
Bukti. Kita tidak dapat menggunakan teorema Ceva untuk membuktikan
teorema ini karena garis-garis sumbu sebuah segitiga bukan merupakan cevian.
Untuk membuktikan teorema ini, kita cukup membuktikan bahwa titik potong
dua buah garis terletak pada garis yang ketiga (Mengapa?).
A

B'

lB

O
lA
B C
A'

Gambar 7.

Misalkan A0 , B 0 , C 0 berturut-turut adalah titik-titik tengah sisi-sisi BC, CA,


AB. Misalkan juga O adalah perpotongan garis lA dan lB . Sekarang akan dibuk-
tikan bahwa O terletak pada garis lC . Pertama, tinjau segitiga OA0 B dan OA0 C.
Dengan teorema Pythagoras pada kedua segitiga tersebut dan karena A0 adalah
titik tengah sisi BC, kita punya
√ √
OB = A0 B 2 + A0 O2 = A0 C 2 + A0 O2 = OC.
Dengan cara yang sama, kita punya bahwa OC = OA. Jadi, kita punya OA = OB,
sehingga OAB segitiga sama kaki. Oleh karena itu, garis tinggi segitiga OAB dari

45
titik O akan memotong titik tengah AB, atau dengan kata lain O terletak pada
garis yang melalui titik tengah AB dan tegak lurus garis AB, yaitu garis lC . Hal
ini melengkapkan pembuktikan.

1.4 Lingkaran Dalam (incircle) dan Lingkaran Luar Segit-


iga (circumcircle)

Lingkaran dalam segitiga adalah lingkaran yang menyinggung ketiga sisi segitiga
dari dalam dan lingkaran luar segitiga adalah lingkaran yang melalui ketiga titik-
titik sudut segitiga. Kita punya beberapa teorema berikut mengenai titik pusat
lingkaran dalam dan luar serta panjang jari-jarinya.

Teorema. Titik perpotongan ketiga garis bagi sudut segitiga ABC adalah
titik pusat lingkaran dalam segitiga ABC dan panjang jari-jarinya sama dengan
[ABC]/s.
Bukti.
A

R
C' Q
B'
I

B A' P C

Gambar 8.

Misalkan AA0 , BB 0 dan CC 0 adalah ketiga garis bagi sudut segitiga ABC yang
berpotongan di titik I. Misalkan juga P , Q, R berturut-turut adalah kaki tegak
lurus titik I pada sisi-sisi BC, CA, AB, atau dengan kata lain, P , Q, R terletak
pada sisi-sisi BC, CA, AB sedemikian hingga IP tegak lurus BC, IQ tegak lurus
CA dan IR tegak lurus AB. Karena ∠QAI = ∠CAA0 = ∠BAA0 = ∠RAI dan
∠AQI = 90◦ = ∠ARI maka kedua segitiga siku-siku AQI dan ARI sebangun.
Kemudian karena sisi miring kedua segitiga siku-siku tersebut berimpit (sehingga
sama panjang), maka kedua segitiga tersebut sebangun. Oleh karenanya, kita
punya bahwa IQ = IR. Dengan cara yang sama, diperoleh juga bahwa IP = IQ.
Jadi kita peroleh IP = IQ = IR. Akibatnya, lingkaran dengan pusat I dan
berjari-jari IP = IQ = IR menyinggung sisi-sisi BC, CA, AB. Lingkaran tersebut

46
kemudian disebut sebagai lingkaran dalam segitiga (incenter) ABC dan jari-jarinya
(IP = IQ = IR) disebut sebagai jari-jari lingkaran dalam segitiga (inradius) ABC.
Sekarang misalkan r menyatakan panjang jari-jari lingkaran dalam segitiga
ABC. Karena segitiga AQI dan ARI kongruen maka AQ = AR dan
1 1
[AQIR] = 2 × [AQI] = 2 × · AQ · IQ = (AQ + AR) · r.
2 2
Dengan cara yang sama, diperoleh
1 1
[BRIP ] = (BR + BP ) · r dan [CP IQ] = (CP + CQ) · r.
2 2
Akibatnya,
[ABC] = [AQIR] + [BRIP ] + [CP IQ]
1 1 1
= (AQ + AR) · r + (BR + BP ) · r + (CP + CQ) · r
2 2 2
1
= (AR + BR + BP + CP + CQ + AQ) · r
2
1
= (AB + BC + CA) · r
2
= sr,
atau setara dengan r = [ABC]/s.
Teorema. Titik perpotongan ketiga garis sumbu segitiga ABC adalah titik
pusat lingkaran luar segitiga ABC dan panjang jari-jarinya sama dengan abc/4[ABC].
Bukti.
A

C' B'

lB

O
lA
B C
A'

47
Gambar 9.

Misalkan lA , lB , lC adalah ketiga garis sumbu segitiga ABC dan O adalah per-
potongan ketiga garis tersebut. Dengan cara yang sama seperti bukti teorema
konkurensi garis sumbu, kita peroleh bahwa OA = OB = OC. Akibatnya, O
adalah titik pusat lingkaran yang melalui ketiga titik sudut segitiga ABC (yaitu
lingkaran yang berpusat di O dengan panjang jari-jari OA = OB = OC). Ling-
karan tersebut selanjutnya dinamakan lingkaran luar segitiga (circumcircle) ABC
dan titik pusatnya disebut titik pusat lingkaran luar segitiga (circumcenter) ABC.

Sekarang misalkan AL adalah sebuah garis tinggi segitiga ABC, sehingga


AL 1
= sin B dan [ABC] = × BC × AL.
AB 2
Jadi,
1 1 1
[ABC] = × BC × AL = × BC × AB sin B = ac sin B.
2 2 2
Selanjutnya dengan aturan sinus, kita punya bahwa
b
= 2R,
sin B
sehingga
1 1 b abc
[ABC] = ac sin B = ac × = ,
2 2 2R 4R
atau setara dengan
abc
R= .
4[ABC]

2 Lingkaran dan Beberapa Sifatnya

2.1 Kuasa Titik Terhadap Lingkaran (Power of a Point


with Respect to a Circle)

Pembahasan pertama di subbab ini adalah tentang hubungan sebuah titik dengan
sebuah lingkaran. Di sini kita mengenal konsep kuasa sebuah titik terhadap sebuah
lingkaran.
Teorema. Diberikan sebuah lingkaran dan sebuah titik P . Buat sebuah garis
yang memotong lingkaran di dua titik A dan B (jika garis tersebut menyinggung
lingkaran, maka A = B). Maka hasil kali P A×P B bernilai tetap, tidak tergantung
oleh garis yang dibuat.

48
Bukti. Buat garis lain yang melalui titik P dan memotong lingkaran di dua
titik C dan D. Sekarang perhatikan bahwa ∠P BC = ∠P DA dan ∠BP C =
∠DP A.
B
A
D A
P
P

C C

Gambar 10.

Dengan demikian, kedua segitiga P BC dan P DA sebangun, sehingga


PB PC
= ,
PD PA
yang setara dengan P A × P B = P C × P D.

Sekarang kita gunakan notasi yang sama dengan notasi pada teorema di atas.
Misalkan O dan r berturut-turut adalah tiik pusat dan panjang jari-jari lingkaran
tersebut. Tinjau garis yang melalui O dan P yang memotong lingkaran di dua
titik R dan S. Nilai
(OP − r)(OP + r) = OP 2 − r2
disebut sebagai kuasa titik P terhadap lingkaran tersebut. Jika P berada di dalam
lingkaran, maka kuasanya negatif, jika P di luar maka kuasanya positif dan jika P
terletak pada lingkaran, kuasanya nol.
R B
A
A

P
P R
O O

S
B

Gambar 11.

49
Terkait dengan teorema sebelumnya, jika P berada di luar atau pada lingkaran,
maka
P A × P B = P R × P S = (OP − r)(OP + r) = OP 2 − r2
sama dengan kuasa P terhadap lingkaran tersebut dan jika P di dalam lingkaran,
maka
P A × P B = P R × P S = (r − OP )(r + OP ) = r2 − OP 2
sama dengan −1 kali kuasa P terhadap lingkaran tersebut.

2.2 Jarak Titik Pusat Lingkaran Dalam dan Luar Segitiga:


Teorema Euler

Teorema kuasa titik terhadap lingkaran di atas dapat digunakan untuk membuk-
tikan salah satu teorema penting dalam geometri: Teorema Euler.

Teorema Euler. Misalkan I dan O berturut-turut adalah titik-titik pusat


lingkaran dalam dan luar segitiga ABC. Jika r dan R berturut-turut menyatakan
panjang jari-jari lingkaran dalam dan luar segitiga ABC, maka

OI 2 = R2 − 2rR.

Bukti.
M
A

α α

I
O
β
β
B α C

Gambar 12.

Misalkan N adalah kaki tegak lurus I pada sisi CA, sehingga IN = r dan
garis bagi sudut AI memotong lingkaran luar segitiga ABC di titik L. Karena
∠BAL = ∠CAL, maka L adalah titik tengah busur BC yang tidak memuat A.

50
Sekarang misalkan garis LO memotong lingkaran luar segitiga ABC lagi di titik
M , sehingga LM adalah diameter lingkaran tersebut yang tegak lurus dengan
BC (Mengapa?). Sekarang misalkan α = 12 A dan β = 12 B. Kita punya bahwa
∠BM L = ∠BAL = α,sehingga
LB LB IN r
= = sin α = = ,
2R LM IA IA
sehingga LB = 2R sin α dan IA = r/ sin α.

Kita juga punya bahwa ∠LBC = ∠LAC = α, sehingga

∠LBI = α + β = 180◦ − ∠AIB = ∠LIB,

sehingga LBI adalah segitiga sama kaki, sehingga LB = LI. Sekarang karena I
berada di dalam lingkaran luar segitiga ABC, maka LI × IA sama dengan −1 kali
kuasa I terhadap lingkaran luar segitiga ABC, yaitu R2 − OI 2 . Di sisi lain, kita
punya bahwa
r
LI × IA = LB × IA = 2R sin α × = 2rR,
sin α
sehingga R2 − OI 2 = 2rR yang setara dengan kesamaan yang ingin dibuktikan.

Sebagai akibat dari teorema tersebut, kita punya ketaksamaan berikut

Akibat. Jika R dan r berturut-turut menyatakan panjang jari-jari lingkaran


luar dan dalam suatu segitiga, maka

R ≥ 2r.

2.3 Segiempat Talibusur dan Beberapa Sifatnya

Segiempat talibusur adalah segiempat yang keempat titik sudutnya terletak pada
satu lingkaran. Berikutnya akan dijelaskan beberapa kriteria dan sifat segiempat
talibusur.

2.3.1 Beberapa Kriteria Segiempat Talibusur

Misalkan ABCD sebuah segiempat talibusur dan O adalah  titik pusat lingkaran
1
luarnya. Kita punya bahwa ∠ADB = ∠ACB = 2 ∠AOB dan ∠ADC+∠ABC =
180◦ . Konvers dari pernyataan tersebut ternyata berlaku. Kita punya teorema
berikut:

51
Teorema. Jika ABCD sebuah segiempat konveks (yaitu kedua diagonalnya
terletak di dalam segiempat) dan ∠ADB = ∠ACB, maka ABCD segiempat tal-
ibusur.
Bukti. Kita menggunakan teknik titik bayangan lagi. Misalkan lingkaran
luar segitiga ABC memotong garis BD di titik D0 (dalam hal ini, D0 adalah titik
bayangan dari titik D). Selanjutnya, kita cukup membuktikan bahwa D0 = D
(Mengapa?).
B
B

A
C
A C
D
D'
D'
D

Gambar 13.

Perhatikan bahwa ABCD0 adalah segiempat talibusur, sehingga kita punya


∠ACB = ∠AD0 B.Dan karena ∠ACB = ∠ADB, maka ∠ADB = ∠AD0 B. Aki-
batnya, D = D0 (Mengapa?) dan kita selesai.

Teorema. Jika ABCD sebuah segiempat dan ∠ADC + ∠ABC = 180◦ , maka
ABCD segiempat talibusur.

Teorema tersebut dapat dibuktikan dengan menggunakan konsep titik bayan-


gan dan pembuktian ini diserahkan kepada pembaca.

Selain dua kriteria dasar di atas, konsep titik bayangan dan teorema kuasa
titik terhadap lingkaran dapat digunakan untuk membuktian kriteria segiempat
talibusur berikut.

Teorema. Misalkan ABCD sebuah segiempat talibusur. Misalkan juga AD


dan BC berpotongan di E dan AB dan CD berpotongan di F . Jika salah satu
dari dua kesamaan berikut:

EA × EC = EB × ED atau F A × F B = F C × F D,

maka ABCD adalah segiempat talibusur.

52
2.3.2 Teorema Ptolemeus dan Brahmagupta

Kita akhiri subbab ini dengan dua teorema tentang segiempat talibusur. Teorema
pertama menyebutkan hubungan antara panjang diagonal dan panjang sisi-sisi
segiempat talibusur dan teorema berikutnya tentang hubungan luas segiempat
talibusur dengan panjang sisi-sisi nya.

Teorema. Misalkan ABCD sebuah segiempat talibusur. Maka

AB × CD + BC × AD = AC × BD.

Bukti.
B C

D
E

Gambar 14.

Misalkan E titik pada segmen AC sehingga ∠ABE = ∠DBC. Karena ∠BAE =


∠BAC = ∠BDC, maka segitiga ABE sebangun dengan segitiga DBC, sehingga
AE CD
=
AB BD
atau setara dengan AE × BD = AB × CD. Sekarang perhatikan bahwa

∠CBE = ∠ABC − ∠ABE = ∠ABC − ∠DBC = ∠DBA

dan ∠BCE = ∠BCA = ∠BDA. Dengan demikian, segitiga BCE sebangun


dengan segitiga BDA, sehingga
CE AD
=
BC BD
atau setara dengan CE · BD = BC · AD. Jadi,

AC × BD = AE × BD + CE × BD = AB × CD + BC × AD.

53
Teorema berikut ini diperoleh oleh Brahmagupta, seorang matematikawan In-
dia pada abad ketujuh A.D.

Teorema. Misalkan ABCD sebuah segiempat talibusur dengan panjang sisi-


sisi AB = a, BC = b, CD = c, DA = d dan s = (a + b + c + d)/2. Maka luas
segiempat talibusur tersebut adalah
p
[ABCD] = (s − a)(s − b)(s − c)(s − d)

Bukti.
A
B
α

π−α
D

Gambar 15.

Salah satu cara termudah membuktikan teorema di atas adalah menggunakan


trigonometri. Misalkan ∠ABC = α, sehingga ∠ADC = 180◦ −α. Sekarang dengan
aturan cosinus pada segitiga ABC dan ADC, kita peroleh

AC 2 = a2 + b2 − 2ab cos α dan AC 2 = c2 + d2 − 2cd cos(180◦ − α).

Karena cos(180◦ − α) = − cos α, dari dua kesamaan di atas kita peroleh

a2 + b2 − 2ab cos α = c2 + d2 + 2cd cos α,

yang setara dengan

2(ab + cd) cos α = a2 + b2 − c2 − d2 ,

sehingga
2
a2 + b2 − c2 − d2

2
cos α =
2(ab + cd)
Sekarang perhatikan bahwa
1 1 1
[ABC] = ab sin α dan [ACD] = cd sin(180◦ − α) = cd sin α,
2 2 2
54
sehingga
1
[ABCD] = [ABC] + [ACD] = (ab + cd) sin α.
2
Dengan demikian
1 1
[ABCD]2 = (ab + cd)2 sin2 α = (ab + cd)2 (1 − cos2 α)
4 4
 2 2 2
!
2 2
1 a + b − c − d
= (ab + cd)2 1 −
4 2(ab + cd)
1 1
= (ab + cd)2 − (a2 + b2 − c2 − d2 )2 .
4 16
Di sisi lain, dengan menjabarkan kedua ruas, kita juga punya bahwa
1 1
(s − a)(s − b)(s − c)(s − d) = (ab + cd)2 − (a2 + b2 − c2 − d2 )2 .
4 16
Kesimpulan selanjutnya mengikuti.

Sebuah kasus khusus dimana titik D berimpit dengan titik A atau C akan
menghasilkan teorema Heron.

Teorema. Misalkan ABC sebuah segitiga dengan panjang sisi-sisi a, b, c dan


s = (a + b + c)/2. Maka
p
[ABC] = s(s − a)(s − b)(s − c).

3 Segiempat

Tidak banyak hal yang bisa dibahas dalam segiempat secara umum. Pada beber-
apa kasus, sebuah segiempat lebih mudah dipandang sebagai dua buah segitiga
yang memiliki satu sisi persekutuan.

3.1 Jajaran Genjang Varignon

Pembahasan pertama ini adalah mengenai bangun datar yang dibentuk oleh titik-
titik tengah keempat sisi sebuah segiempat. Kita memiliki teorema berikut:

Teorema. Jika K, L, M , N adalah titik-titik tengah sisi-sisi AB, BC, CD,


DA pada segiempat ABCD, maka KLM N adalah jajaran genjang. Jajaran gen-
jang yang terbentuk tersebut disebut sebagai jajaran genjang Varignon. Lebih
jauh, luas jajaran genjang Varignon sama dengan setengah luas segiempatnya.

55
Bukti.
B

L
A

N
C
M
D

Gambar 16.

Perhatikan segitiga ABC. Karena K dan L berturut-turut adalah titik-titik


tengah AB dan BC, maka KL tegak lurus AC. Dengan cara yang sama, kita
punya bahwa M N juga tegak lurus dengan AC. Akibatnya, KL sejajar dengan
AC. Dengan cara yang sama juga, kita peroleh KN sejajar LM . Kesimpulan
mengikuti dengan mudah.

Untuk luasnya, kita punya bahwa

[KLM N ] = [ABCD] − [AKN ] − [BKL] − [CLM ] − [DM N ]


1 1 1 1
= [ABCD] − [ABD] − [ABC] − [BCD] − [CDA]
4 4 4 4
1 1
= [ABCD] − [ABCD] − [ABCD]
4 4
1
= [ABCD].
2

3.2 Lingkaran Dalam Segiempat

Tidak semua segiempat memiliki lingkaran dalam, yaitu suatu lingkaran di dalam
segiempat yang menyinggung keempat sisi segiempat. Teorema berikut mem-
berikan sifat sebuah segiempat yang memiliki lingkaran dalam

Teorema. Misalkan ABCD adalah segiempat konveks yang memiliki lingkaran


dalam. Maka AB + CD = BC + DA.
Bukti. Misalkan lingkaran dalam segiempat ABCD menyinggung sisi-sisi AB,
BC, CD, DA berturut-turut di titik-titik K, L, M , N .

56
B
K
A

N
L

D
M C

Gambar 17.

Dengan demikian, kita punya

AK = AN , BK = BL, CL = CM , dan DM = DN

sehingga

AB + CD = AK + BK + CM + DM = AN + BL + CL + DN = BC + DA.

Konvers dari teorama di atas ternyata berlaku. Kita punya teorema berikut.

Teorema. Misalkan ABCD adalah segiempat konveks yang memenuhi AB +


CD = BC + DA. Maka ABCD memiliki lingkaran dalam.
Bukti.
B
K
A

P
L
D
D'
M

Gambar 18.

57
Misalkan garis AB dan CD berpotongan di titik P dan Γ adalah lingkaran
dalam segitiga P BC. Misalkan juga garis melalui A yang menyinggung Γ mem-
otong P C di titik D0 sehingga Γ juga merupakan lingkaran dalam segiempat
ABCD0 . Selanjutnya, cukup dibuktikan bahwa D = D0 .

Dengan menggunakan teorema sebelumnya, kita punya AB +CD0 = BC +D0 A


dan menggunakan kesamaan yang diberikan, kita punya
CD0 − CD = AB + CD0 − (AB + CD)
= BC + D0 A − (BC + DA) = D0 A − DA.
Jika CD0 > CD, maka DD0 = CD0 − CD = D0 A − DA, sehingga D0 A =
DA + DD0 yang jelas tidak mungkin karena menurut ketaksamaan segitiga D0 A <
DA + DD0 . Demikian juga halnya dengan CD0 < CD. Kita simpulkan bahwa
CD0 = CD, sehingga D = D0 dan kita selesai.

4 Lampiran

4.1 Rumus-Rumus Trigonometri

Berikut diberikan rangkuman rumus-rumus trigonometri terutama untuk sinus dan


cosinus. Rumus untuk tangent dan cotangent dapat diturunkan dari rumus-rumus
tersebut. Pertama adalah rumus trigonometri untuk jumlah maupun selisih dua
sudut dan rumus untuk mengubah jumlah atau selisih sinus dan cosinus menjadi
hasil kali sinus dan cosinus.

cos(A + B) = cos A cos B − sin A sin B


cos(A − B) = cos A cos B + sin A sin B
sin(A + B) = sin A cos B + cos A sin B
sin(A − B) = sin A cos B − cos A sin B
   
A+B A−B
sin A + sin B = 2 sin cos
2 2
   
A+B A−B
sin A − sin B = 2 cos sin
2 2
   
A+B A−B
cos A + cos B = 2 sin cos
2 2
   
A+B A−B
cos A − cos B = −2 sin sin .
2 2

58
Selanjutnya adalah rumus trigonometri untuk dua kali sudut atau setengah kali
sudut.

sin 2A = 2 sin A cos A


cos 2A = 2 cos2 A − 1 = 1 − 2 sin2 A
2 tan A
tan 2A =
1r− tan2 A
A 1 − cos A
sin = ±
2 r 2
A 1 + cos A
cos = ± .
2 2

4.2 Perluasan Aturan Sinus dan Aturan Cosinus

Teorema (Aturan Sinus). Misalkan ABC segitiga dengan panjang jari-jari


lingkaran luar R. Maka
a b c
= = = 2R.
sin A sin B sin C

Teorema (Aturan Cosinus). Misalkan ABC segitiga. Maka

a2 = b2 + c2 − 2bc cos A
b2 = c2 + a2 − 2ca cos B
c2 = a2 + b2 − 2ab cos C.

59
Edited by Foxit Reader
Copyright(C) by Foxit Corporation,2005-2009
For Evaluation Only.

KOMBINATORIKA
Utari Wijayanti

1. Permutasi dan Kombinasi


1.1 Faktorial
Kita definisikan simbol ! (faktorial), sebagai berikut 0! = 1, dan untuk bilangan asli n1,

n! = 1 · 2 ··· n.

Perhatikan n! dibaca sebagai n faktorial.

Contoh 1 Berdasarkan definisi di atas, kita dapatkan


1! = 1
2! = 1 · 2 = 2,
3! = 1 · 2 · 3 = 6,
4! = 1 · 2 · 3 · 4 = 24,
5! = 1 · 2 · 3 · 4 · 5 = 120.

Contoh 2 Perhatikan bahwa


7! 7.6.5.4!
210
4! 4!

2 ! 2 1 !
2 1
! !

2 ! 2 ! 1
1 ! 1 1 2 ! 1 1

1.2 Permutasi Tanpa Pengulangan


Definisi Misalkan x1, x2, . . . , xn merupakan n obyek berbeda. Suatu permutasi dari obyek-
obyek ini merupakan penyusunan obyek-obyek tersebut dengan memperhatikan urutan.
Untuk selanjutnya permutasi n obyek disebut sebagai n-permutasi.

Contoh 3 Terdapat 24 permutasi untuk huruf-huruf pada kata MATH, yakni


MATH, MAHT, MTAH, MTHA, MHAT, MHTA,
AMTH, AMHT, ATMH, ATHM, AHMT, AHTM,
TAMH, TAHM, TMAH, TMHA, THAM, THMA,
HAMT, HATM, HMAT, HMTA, HTAM, HTMA.

60
Teorema Misalkan x1, x2, … , xn adalah n obyek yang berbeda. Maka banyaknya n −
permutasi adalah n! permutasi.

Contoh 4 Banyaknya permutasi dari huruf-huruf dari kata RAMBUT adalah 6! = 720.

Contoh 5 Sebuah lemari memuat 5 buku berbahasa Jerman, 7 buku berbahasa Spanyol, dan 8
buku berbahasa Indonesia. Diketahui bahwa tidak ada dua buku yang sama.
a. Berapa banyak penyusunan berbeda yang bisa dilakukan pada buku-buku ini?
b. Berapa banyak penyusunan berbeda yang bisa dilakukan pada buku-buku ini jika
buku-buku dari masing-masing bahasa harus diletakkan saling berdekatan?
c. Berapa banyak penyusunan berbeda yang bisa dilakukan pada buku-buku ini jika
semua buku berbahasa Indonesia harus saling berdekatan?
d. Berapa banyak penyusunan berbeda yang bisa dilakukan pada buku-buku ini jika
tidak boleh ada dua buku berbahasa Indonesia diletakkan saling berdekatan?

Jawab:
a. Perhatikan bahwa kita melakukan permutasi pada 5+7+8 = 20 buku. Maka banyaknya
kemungkinan penyusunan adalah 20!.
b. Buku-buku dengan bahasa yang sama di’ikat’ sehingga saling berdekatan. Perhatikan
bahwa kita melakukan permutasi pada 3 bahasa, yakni dengan 3! cara. Kemudian, kita
lakukan permutasi pada buku-buku berbasa Jerman dalam 5! cara, pada buku-buku
berbahasa Spanyol dalam 7! cara, dan pada buku-buku berbahasa Indonesia dengan 8!
cara. Jadi total banyaknya cara penyusunan buku adalah 3!5!7!8!.
c. Pertama-tama, kita atur dulu buku-buku berbahasa Jerman dan Spanyol. Perhatikan
dengan mengatur 5 + 7 = 12 buku-buku ini, kita mendapatkan 13 ’ruang’, yakni ruang
sebelum buku pertama, ruang di antara buku-buku, dan ruang setelah buku terakhir.
Untuk memastikan bahwa buku-buku berbahasa Indonesia saling berdekatan, buku-
buku ini kita ’ikat’ bersama-sama dan diletakkan pada salah satu ’ruang’. Perhatikan
bahwa buku-buku berbahasa Indonesia dapat disusun dengan 8! cara, sedangkan
buku-buku berbahasa Jerman dan Perancis dengan 12! cara. Sehingga total banyaknya
cara adalah 13⋅8!12!.
d. Pertama-tama atur buku berbahasa Jerman dan Spanyol. Dengan mengatur 5 + 7 = 12
buku ini, diperoleh 13 ’ruang’. Untuk memastikan bahwa tidak ada buku-buku
berbahasa Indonesia yang saling berdekatan, kita letakkan buku-buku tersebut ke
dalam ’ruang-ruang’ ini. Buku pertama dapat ditempatkan ke salah satu dari 13 ruang.
Buku kedua dapat ditempatkan ke sisa 12 ruang yang ada, buku ketiga dapat
ditempatkan ke sisa 11 ruang yang ada, dan seterusnya, hingga buku ke 8 dapat
ditempatkan ke sisa 6 ruang yang ada. Jadi total banyaknya pengaturan yang ada
adalah 13 · 12 · 11 · 10 · 9 · 8 · 7 · 6 · 12!.

Latihan 6 Sebuah rak buku terdiri atas 3 novel Rusia, 4 novel Jerman, dan 5 novel Indonesia.
Ada berapa banyak cara penyusunan novel-novel ini jika,

a. tidak ada batasan pengelompokkan novel


b. semua novel Indonesia harus berdekatan
c. tidak ada novel Indonesia yang saling berdekatan

Latihan 7 Berapa banyak kemungkinan permutasi untuk kata LEMARI? Berapakah banyak
permutasi yang diawali dengan huruf R dan diakhiri dengan huruf E? Berapa banyak

61
permutasi jika P dan U selalu bersama-sama dengan urutan PU? Berapa banyak permutasi
yang ada jika tidak ada huruf vokal (A, E, I) yang berdekatan?

Latihan 8 Berapa banyak pengaturan yang bisa dilakukan pada huruf-huruf dari kata
TERANG jika huruf N dan G tidak pernah terpisah?

Latihan 9 (AIME 1991) Diketahui sebuah bilangan rasional, tuliskan sebagai perbandingan
paling sederhana, kemudian kalikan penyebut dengan pembilang. Tentukan berhapa banyak
bilangan rasional yang nilainya di antara 0 dan 1 dimana 20! menjadi hasil kalinya?

Latihan 10 Seekor laba-laba mempunyai sebuah kaos kaki dan sebuah sepatu untuk masing-
masing dari delapan kakinya. Tentukan banyaknya kemungkinan urutan laba-laba bisa
menggunakan kaos kaki dan sepatu, dengan asumsi bahwa, untuk setiap kaki, kaos kaki harus
digunakan sebelum sepatu?

Latihan 11 Ada berapa banyak carakah 8 orang bisa ditempatkan dalam sebuah kursi
panjang, jika:
a. tidak ada batasan dalam pengaturan posisi duduk
b. orang X dan Y harus duduk berdampingan
c. ada 4 pria dan 4 wanita dimana tidak ada 2 pria atau 2 wanita yang duduk
berdampingan
d. ada 4 pasang suami-istri di mana setiap pasang suami-istri harus duduk berdampingan
e. ada 4 pria dan mereka harus duduk berdekatan.

1.3 Permutasi dengan Pengulangan

Sekarang kita akan membahas permutasi di mana sebuah elemen/obyek dapat dipilih lebih
dari satu kali.

Contoh 12. Ada berapa banyak carakah huruf-huruf dari kata LALALILILU dapat
dipermutasi?
Jawab: Misalkan kita memberi index pada setiap huruf yang berulang sehingga menjadi
L1A1L2A2L3I1L4I2L5U.

Maka sekarang terdapat 10 obyek, yang dapat dipermutasi dengan 10! cara yang berbeda.
Untuk setiap 10! permutasi ini, huruf A1A2 dapat dipermutasi dengan 2! cara, L1L2L3L4L5
dapat dipermutasi dengan 5! cara, dan huruf I1I2 dapat dipermutasi dengan 2! cara. Jadi
sebenarnya 10! terlalu banyak terhitung, dan dapat diperbaiki menjadi
10!
2! 5! 2!

Teorema Misalkan terdapat k tipe obyek, dengan: n1 obyek bertipe 1, n2 obyek bertipe 2, dst.
Maka banyaknya cara penyusunan dari n1+n2+. . .+nk obyek ini adalah dengan
! !… !

Contoh 13. Ada berapa banyak cara huruf-huruf dari kata SUMSUM bisa disusun
sedemikian hingga tidak ada dua huruf yang sama berdekatan?

62
Jawab: Misalkan kita telah memilih SU pada dua posisi pertama, maka susunan yang
mungkin bagi huruf M adalah sebagai berikut:
S U M M

S U M M

S U M M

Pada kasus yang pertama, terdapat 2! =2 kemungkinan penyusunan huruf S dan U, pada
kasus kedua ada 2!=2 kemungkinan, sedangkan pada kemungkinan ketiga hanya ada 1
kemungkinan. Jadi jika kata yang disusun dimulai dengan huruf S dan U, maka
memunculkan 2+2+1=5 kemungkinan penyusunan. Secara umum, kita bisa memilih 3 huruf
untuk posisi pertama, dan 2 huruf untuk posisi kedua. Sehingga banyaknya kemungkinan
yang dicari adalah 3 ⋅ 2 ⋅ 5 =30.

Latihan 14. Ada berapa banyak bilangan yang bisa dibentuk oleh angka 1, 2, 3, 4, 3, 2, 1
sehingga angka ganjil menempati posisi ganjil?

1.4 Kombinasi tanpa Pengulangan

Misalkan diberikan sebuah himpunan dengan n elemen. Pemilihan k anggota himpunan


tanpa memperhatikan urutan disebut k-kombinasi, dengan 0 ≤ k ≤ n. Jadi k-kombinasi adalah
sebuah subhimpunan dengan k anggota (k-subhimpunan).

Banyaknya cara untuk memilih r-kombinasi dari sebuah himpunan dengan n anggota
dinotasikan dengan . Dengan demikian menyatakan banyaknya k-subhimpunan dari
!
himpunan dengan n anggota. Dapat ditunjukkan bahwa
! !

Catatan: Perhatikan bahwa 1 sedangkan .


0 1 1
Karena n - (n - k) = k , maka untuk untuk bilangan bulat n, k berlaku sifat identitas simetri,
yakni

! !
! ! ! !

Hal ini bisa diinterpretasikan sebagai berikut: jika ada n pinsil warna yang berbeda, maka
banyaknya cara memilih k pinsil sama dengan banyaknya cara memilih n-k pinsil yang tidak
diambil.

11 11
Contoh15. 55
9 2

63
12 12
792.
5 7

Contoh 16. 2-kombinasi dari himpunan {P, Q, R, S} adalah

PQ, PR, PS, QR, QS, RS

Contoh 17. 3-kombinasi dari himpunan {P, Q, R, S} adalah

PQR, PQS, PRS, QRS

10
Contoh 18. Dari 10 buku tulis, kita bisa memilih 4 dengan menggunakan 210 cara.
4

1.5 Kombinasi dengan Pengulangan

Teorema (De Moivre) Misalkan n adalah bilangan asli. Banyaknya solusi bilangan asli yang
memenuhi

x 1 + x 2 + · · · + xr = n
adalah
1

Bukti: Tuliskan n sebagai


n =1 + 1 + … + 1,
dengan n angka 1, dan n -1 notasi + . Untuk mendekomposi n menjadi penjumlahan r
bilangan kita hanya perlu memilih r – 1 notasi + , yang membuktikan teorema.

Contoh 19. Berapa banyak solusi bilangan bulat non-negatif untuk persamaan

x1 + x2 + x3 + x4=11

Jawab: Contoh ini bisa diselesaikan dengan secara langsung menggunakan Teorema De
Moivre. Tetapi berikut ini akan diberikan ilustrasi sehingga bisa diperoleh gambaran lebih
jelas.

Perhatikan barisan dari 14 (11+3) bintang di bawah ini.

**************

64
Pilih tiga dari 14 bintang ini, dan ubah menjadi garis tegak. Dengan demikian, ketiga garis
tegak ini membagi 11 bintang menjadi 4 kelompok (sebuah kelompok dimungkinkan untuk
tidak memiliki anggota). Setiap pilihan 3 garis tegak berpadanan tepat dengan satu solusi
persamaan di atas, dengan memandang banyaknya bintang yang terletak dalam suatu
kelompok sebagai nilai dari sebuah variable. Sebagai contoh, susunan di bawah ini
berpadanan dengan solusi

x1=1, x2=0, x3=5, x4=5.

* || *****|*****

Dengan demikian banyaknya solusi dari persamaan di atas adalah

11 4 1 14 14
364
4 1 3 11

Contoh 20. Tentukan banyaknya lintasan terpendek dari A ke B.


B

Jawab: Setiap lintasan terpendek dari A ke B harus terdiri dari 9 langkah, di mana 4 di
antaranya adalah langkah ke atas dan sisanya langkah ke kanan. Dengan demikian
9 9
banyaknya lintasan terpendek adalah 126.
4 5

Contoh 21. Misalkan terdapat sepuluh bola yang identik, dan keranjang yang dinomori 1, 2,
…, 8. Tentukan banyaknya cara untuk mendistribusikan bola ke dalam keranjang agar setiap
keranjang terisi sedikitnya 1 bola.

Jawab: Banyaknya cara adalah


10 1 9
36.
8 1 7

Jelaskan mengapa demikian!

65
2. Prinsip Sangkar Merpati (Pigeon-hole Principle)
Prinsip sangkar merpati menyatakan bahwa jika terdapat n barang yang didistribusikan ke
dalam n buah kotak, maka sedikitnya satu kotak akan menerima lebih dari satu barang.

Untuk menghemat penulisan, pada contoh-contoh berikut prinsip ini ditulis sebagai PHP.

Contoh 1. Jika terdapat 13 orang, maka sedikitnya ada dua orang merayakan ulang tahun
pada bulan yang sama

Contoh 2. kemudian jika rata-rata manusia memiliki dua juta rambut, maka setidaknya ada
empat orang di Jakarta memiliki jumlah rambut yang sama di kepalanya.

Perhatikan bahwa prinsip sangkar merpati bermanfaat dalam membuktikan masalah


eksistensi, yakni kita menunjukkan sesuatu ‘ada’ tanpa perlu diidentifikasi secara kongkret.

Contoh 3. (Putnam 1978) Misalkan A adalah himpunan dua puluh bilangan asli yang dipilih
dari
deret matematika 1, 4, . . . , 100. Buktikan bahwa ada dua bilangan asli berbeda dalam A
yang jumlahnya 104.

Jawab: Kita partisi ketigapuluh empat anggota dari deret ini menjadi delapan belas himpunan,
yakni {1}, {52}, {4, 100}, {7, 97}, {10, 94}, . . . , {49, 55}. Karena kita memilih dua puluh
bilangan asli dan kita mempunyai delapan belas himpunan, maka berdasarkan PHP, terdapat
dua bilangan yang merupakan anggota dari himpunan yang sama. Perhatikan bahwa
jumlah keduanya adalah 104.

Contoh 4. Tunjukkan bahwa di antara tujuh bilangan asli positif yang nilainya kurang atau
sama dengan 126, bisa kita temukan dua diantaranya, katakan a dan b, yang memenuhi b < a
≤ 2b.

Jawab: Bagi bilangan-bilangan {1, 2, 3, . . . , 126} menjadi enam himpunan berikut:


{1, 2}, {3, 4, 5, 6}, {7, 8, . . . , 13, 14}, {15, 16, . . . , 29, 30},
{31, 32, . . . , 61, 62} dan {63, 64, . . . , 126}.

Berdasarkan PHP, dua dari tujuh bilangan pasti terletak dalam salah satu himpunan, dan jelas
bahwa kedua bilangan tersebut akan memenuhi ketidaksamaan yang diminta.

Contoh 5. Buktikan bahwa bagaimanapun lima puluh lima bilangan dipilih dari {1, 2, . . . ,
100}, pasti terdapat dua bilangan yang selisihnya 10.

Jawab: Pertama-tama perhatikan bahwa jika kita memilih n + 1 bilangan dari sebarang 2n
bilangan asli berurutan, maka terdapat dua bilangan yang selisihnya n. Karena jika kita
memasangkan 2n bilangan asli berurutan {a + 1, a + 2, . . . , a + 2n} menjadi n pasang

66
{a + 1, a + n + 1}, {a + 2, a + n + 2}, . . . , {a + n, a+2n}, kemudian kita pilih n + 1 bilangan,
maka berdasarkan PHP terdapat dua bilangan berada dalam kelompok yang sama. Sekarang,
kelompokkan 100 bilangan menjadi:
{1, 2, . . . , 20}, {21, 22, . . . , 40},
{41, 42, . . . , 60}, {61, 62, . . . , 80} dan {81, 82, . . . , 100}.

Jika kita memilih limapuluh lima bilangan, maka berdasarkan PHP, pasti terdapat suatu
kelompok dimana kita memilih sedikitnya sebelas bilangan. Dengan menerapkan lemma
sebelumnya pada kelompok tersebut (yakni untuk n = 10), maka terdapat dua bilangan
dengan selisih 10.

Contoh 6. (AHSME 1994) Namai sebuah CD dengan label ”1”, dua CD dengan label ”2”,
tiga CD dengan label ”3”, . . ., dan lima puluh CD dengan label ”50”. Kemudian letakkan 1 +
2 + 3 + . . . + 50 = 1275 CD berlabel ini ke dalam suatu kotak. Beberapa CD diambil dari
kotak secara acak tanpa penggantian. Berapakah jumlah minimum CD yang harus diambil
untuk menjamin diperoleh setidaknya sepuluh CD dengan label yang sama?

Jawab: Jika kita mengambil semua CD yang berlabel ”1”,. . .,”9” (ada 1+2+. . .+9 = 45 CD)
dan sembilan CD dari setiap CD yang berlabel ”10”, . . ., ”50”, maka kita telah mengambil
45+9 ·41 = 414 CD. Maka pengambilan CD ke-415 akan memastikan terdapat setidaknya 10
CD dengan label yang sama.

Contoh 7. (IMO 1964) Tujuhbelas orang saling berkorespondensi melalui email. Setiap
orang berkorespondensi dengan keenambelas orang lainnya, dan diketahui mereka hanya
membahas tiga topik yang berbeda. Jika setiap pasang korespondensi hanya membahas
sebuah topik, buktikan bahwa terdapat setidaknya tiga orang saling berkorespondensi
dengan topik yang sama.

Jawab: Pilih seorang dalam kelompok tersebut, misalkan Candra. Perhatikan bahwa ia
berkorespondensi dengan enam belas orang lainnya. Berdasarkan PHP, setidaknya ada enam
orang yang berkorespondensi dengan Candra dan membahas topik yang sama, sebut topik
tersebut sebagai topik A. Jika di antara enam orang tersebut ada dua orang yang membahas
topik A juga, maka masalah selesai. Karena itu berarti Candra dan mereka berdua saling
berkorespondensi dengan topic yang sama, yakni topik A.

Jika di antara enam orang tersebut tidak ada dua orang yang saling berkorespondensi dengan
topik A, maka keenam orang tersebut hanya boleh saling berkorespondensi dengan dua topik
lainnya, sebut sebagai topik B dan C. Pilih seorang dari enam orang ini, misalkan Budi.
Berdasarkan PHP, di antara lima teman korespondensi Budi ini, pasti terdapat tiga
diantaranya yang saling berkorespondensi dengan topic yang sama. Tanpa mengurangi
keumuman, misalkan topik B. Jika ada dua dari tiga orang ini yang saling berkorespondensi
dengan topik B juga, maka masalah selesai. Tetapi jika tidak, maka setiap dua dari tiga orang
ini saling berkorespondensi dengan topik C, dan artinya mereka bertiga saling
berkorespondensi dengan topik yang sama, yakni topic C. Terbukti.

Contoh 8. Diketahui A suatu himpunan dengan anggota sepuluh bilangan asli yang bernilai
di antara 1 dan 99. Buktikan bahwa terdapat dua subhimpunan yang irisannya bukan
himpunan kosong dimana jumlah anggota-anggota kedua subhimpunan adalah sama.

67
Jawab: Dari suatu himpunan dengan 10 anggota, kita dapat memperoleh 210−1 = 1023
subhimpunan tidak kosong. Untuk setiap subhimpunan ini, kita tentukan jumlah dari anggota-
anggotanya. Perhatikan bahwa nilai maksimal yang mungkin kita peroleh dari jumlah
anggota suatu subhimpunan adalah 90 + 91 + . . . + 99 = 945 < 1023. Akibatnya, berdasarkan
PHP, terdapat setidaknya dua subhimpunan yang memiliki jumlah anggota sama.

Contoh 9. Diberikan 9 bilangan asli sebarang yang faktor primanya anggota dari himpunan
{3, 7, 11}. Buktikan bahwa terdapat setidaknya dua bilangan sehingga hasil kalinya
merupakan bilangan kuadrat.

Jawab: Suatu bilangan asli merupaka,n bilangan kuadrat jika semua pangkat dari faktor
primanya merupakan bilangan genap. Perhatikan bahwa semua bilangan asli tersebut dapat
dinyatakan sebagai 3a7b11c. Setiap triplet (a, b, c) memiliki salah satu bentuk dari 8 pola
paritas berikut: (genap,genap,genap), (genap,genap,ganjil), (genap,ganjil,genap),
(genap,ganjil,ganjil), (ganjil,genap,genap), ..., (ganjil, genap, ganjil), (ganjil, ganjil, genap),
(ganjil,ganjil,ganjil). Karena terdapat 9 bilangan asli, maka berdasarkan PHP, terdapat dua
bilangan yang triplet pangkatnya memiliki paritas yang sama. Pilih dua bilangan ini, maka
hasil kalinya merupakan bilangan kuadrat, karena jumlah dari tiap pangkat yang bersesuaian
merupakan bilangan genap.

LATIHAN

Latihan 10. Buktikan bahwa di antara n + 1 bilangan, pasti terdapat setidaknya dua bilangan
yang selisihnya habis dibagi n.

Latihan 10. Tunjukkan bahwa jika lima titik sebarang semuanya pada atau di dalam suatu
persegi dengan panjang sisi 1, maka terdapat sepasang titik yang jaraknya kurang atau sama
dengan √2/2.

Latihan 12. (Hungarian Math Olympiad, 1947) Buktikan bahwa di antara enam orang dalam
suatu ruangan terdapat tiga orang yang saling kenal, atau terdapat setidaknya tiga orang yang
tidak saling kenal.

Latihan 13. Kita menyebut suatu himpunan ”bebas jumlah” jika tidak ada dua anggota
himpunan yang jumlahnya adalah anggota dari himpunan itu juga. Berapa maksimal
banyaknya anggota subhimpunan dari himpunan {1, 2, . . . , 2n − 1} yang ”bebas jumlah”?

Petunjuk: Perhatikan bahwa himpunan {n, n+1,n+2,. . . , 2n-1} dengan n anggota adalah
”bebas jumlah”. Tunjukkan bahwa sebarang subhimpunan dengan n + 1 anggota tidak bebas
jumlah.
Latihan 14. (MMPC 1992) Misalkan huruf-huruf alfabet dituliskan secara acak.
a. Buktikan bahwa pasti terdapat empat huruf konsonan yang berurutan
b. Tuliskan suatu daftar huruf alfabet dimana tidak terdapat lima huruf konsonan
berurutan.
c. Buktikan bahwa jika semua huruf diatur secara melingkar, maka terdapat lima huruf
konsonan berurutan.

Latihan 15. Misalkan M adalah bilangan asli tujuh belas angka dan misalkan N adalah
bilangan yang diperoleh dari M dengan menuliskan angka-angka yang sama tapi dengan

68
urutan terbalik. Buktikan bahwa terdapat setidaknya satu angka dari representasi bilangan M
+N yang genap.

Latihan 16. Tidak peduli bagaimanapun lima puluh lima bilangan asli dipilih dari {1, 2, . . . ,
100}, buktikan bahwa dapat dipilih dua bilangan yang memiliki selisih 9, dua yang memiliki
selisih 10, dua yang memiliki selisih 12, dan 2 yang memiliki selisih 13, tetapi tidak harus
ada dua yang memiliki selisih 11.

Latihan 17. Diketahui mn + 1 bilangan real yang berbeda. Buktikan bahwa terdapat suatu
barisan meningkat dengan setidaknya n + 1 anggota, atau barisan menurun dengan setidaknya
m + 1 anggota.

Latihan 18. Jika titik-titik pada suatu bidang diwarnai dengan tiga warna, tunjukkan bahwa
akan selalu terdapat dua titik dengan warna yang sama berjarak satu satuan.

Latihan 19. Tunjukkan bahwa jika titik-titik ada suatu bidang diwarnai dengan dua warna,
maka akan selalu terdapat sebuah segitiga sama sisi yang titik sudut-titik sudutnya memiliki
warna yang sama.

3. Paritas
Contoh 1. Misalkan dua persegi di sudut yang berseberangan dari sebuah papan catur
dibuang. Tunjukkan bahwa tidak mungkin 62 persegi yang tersisa ditutup oleh 31 domino.

Jawab: Setiap domino menutup tepat satu kotak putih dan satu kotak hitam. Tetapi dua kotak
persegi yang dibuang berwarna sama, sehingga jelas bahwa tidak mungkin persegi yang
tersisa ditutup dengan 31 domino.

Contoh 2. Bilangan 1, 2, …, 10 dituliskan dalam suatu baris. Tunjukkan bahwa


bagaimanapun tanda penjumlahan atau pengurangan digunakan di depan masing-masing
bilangan, maka jumlahnya tidak akan pernah menjadi 0.

Jawab: Jumlah dari 1 + 2 + … + 10 = 55, suatu bilangan ganjil. Karena paritas tidak
dipengaruhi oleh pemilihan tanda penjumlahan atau pengurangan, maka bagaimanpun tanda
di depan masing-masing angka 1, 2, …, 10 dipilih, jumlahnya tidak mungkin menjadi 0.

Definisi. Titik latis (m, n) pada suatu bidang adalah titik dengan koordinat bilangan bulat.

Definisi. Titik tengah dari suatu ruas garis yang menghubungkan (x,y) ke (x1,y1) adalah titik
,
2 2

Contoh3 . Lima titik latis dipilih secara acak. Buktikan bahwa kita selalu bisa memilih dua
titik sehingga titik tengah yang menghubungkan keduanya juga merupakan titik latis.

Jawab: Perhatikan bahwa terdapat empat pola paritas: (genap, genap), (genap, ganjil),
(ganjil, ganjil), (ganjil, genap). Berdasarkan PHP di antara lima titik latis pasti terdapat dua

69
titik yang memiliki paritas yang sama. Pilih dua, jelas bahwa titik tengahnya merupakan
bilangan bulat juga.

Untuk contoh-contoh berikut, kita perlu mengetahui nama-nama dari tetromino berikut:

Contoh 4. Budi mengambil setiap potong dari masing-masing tetromino pada gambar di
atas. Tunjukkan bahwa bagaimanapun penyusunan dilakukan, tidak mungkin dihasilkan
sebuah persegi panjang.

Jawab: Jika penyusunan sebuah persegi panjang dimungkinkan, maka akan terdiri dari 20
persegi. Warnai persegi-persegi ini sebagaimana pewarnaan pada papan catur. Maka
terdapat 10 persegi berwarna hitam dan 10 berwarna putih. Perhatikan bahwa T-tetromino
selalu menutupi persegi berwarna hitam dengan jumlah ganjil, sedangkan tetromino lainnya
selalu menutupi dengan jumlah genap. Sehingga banyaknya persegi warna hitam yang
tertutupi adalah ganjil. Kontradiksi.

Contoh 5. Tunjukkan bahwa sebuah papan catur 8×8 tidak bisa ditutupi dengan 15 straight-
tetromino dan sebuah L-tetromino.

Jawab: Warnai baris 1, 3, 5, 7 dengan warna hitam dan baris 2, 4, 6, 8 dengan warna putih.
Straight-tetromino akan selalu menutupi persegi putih dengan jumlah genap, sedangkan L-
tetromino akan selalu menutupi dengan jumlah ganjil. Jika dimungkinkan ada pengubinan
yang diminta, maka kita akan menutupi ubin warna putih dengan jumlah ganjil, kontradiksi.

Latihan 6. Dua puluh anak laki-laki dan perempuan duduk di sekeliling meja bundar.
Tunjukkan bahwa pasti terdapat seseorang yang diapit oleh dua orang anak perempuan.

Latihan 7. Angka 1, 2, …, 2001 dituliskan pada papan. Seseorang menghapuskan sepasang


bilangan pada papan kemudian menuliskan selisihnya di papan. Hal tersebut dilakukan
berulang kali hingga tinggal tersisa sebuah bilangan. Apakah mungkin bilangan tersebut 0?

Latihan 8. Tunjukkan bahwa sebuah papan catur berukuran 10×10 tidak bisa ditutupi dengan
25 straight tetromino.

Latihan 9. Tunjukkan bahwa sebuah papan catur berukuran 8×8 tidak bisa ditutupi dengan
15 T-tetromino dan sebuah tetromino.

Latihan 10. Sebuah pertemuan dihadiri oleh n peserta. Sejumlah peserta saling berjabat
tangan. Tida ada yang berjabat tangan dengan dirinya sendiri dan setiap dua peserta berjabat
tangan paling banyak satu kali. Seseorang peserta dikatakan ganjil jika banyaknya jabat
tangan yang dilakukannya adalah ganjil. Tunjukkan bahwa banyaknya peserta ganjil adalah
genap.

70
Prinsip ini digunakan untuk menentukan kardinalitas dari gabungan himpunan-himpunan yang tidak
harus saling lepas. Untuk kasus di mana banyaknya himpunan adalah dua atau tiga, dengan
menggunakan diagram Venn kita dapat dengan mudah menunjukkan bahwa

|A | | | | | | |,

dan

| | | | | | | | | | | | | | | |.

Secara umum, jika diberikan n buah himpunan maka kardinalitas dari gabungan n buah
himpunan ini diberikan oleh

| … |

| | 1

Contoh 1. Pada sebuah klub olahraga 10 orang menyukai tenis, 15 orang menyukai squash,
12 orang menyukai badminton. Di antara mereka, 5 orang menyukai tenis dan squash, 4
orang menyukai tenis dan badminton, 3 orang menyukai squash dan badminton, dan 2 orang
menyukai ketiga olahraga ini. Berapa banyak anggota klub yang menyukai sedikitnya satu
dari ketiga cabang olahraga ini?

Jawab: Misalkan T, S, B, secara berturut-turut, adalah himpunan anggota klub yang


menyukai tenis, squash dan badminton. Maka

| | | | | | | | | | | | | | | |
= 10 + 15 + 12 – 5 – 4 – 3 + 2
= 27 .

71

You might also like